SlideShare a Scribd company logo
1 of 239
Womens Health ( MRCGP Qs ) : Contraception
Q1)_An 18-year-old female presentstoher GP as she has missedone of her Microgynon30 pills
yesterdaymorning. She has taken Microgynonfor the past 2 years and is currently 4 days intoa
packet of pills.She had sexual intercourse last nightand isunsure what to do. What is the correct
management?
A. Advise condomuse fornext7 days
B. Performa pregnancytest
C. Omitpill breakat endof pack
D. No action needed
E. Emergencycontraceptionshouldbe offered
As she has only missed one pill no action is needed. For further information please consult the link
to the FSRH guidelines.
The October 2011 AKT feedback stated: 'With regard to AKT 13, knowledge about basic
contraceptive issues seemed to be lacking. '
Combined oral contraceptive pill: missed pill
The advice from the Faculty of Sexual and Reproductive Healthcare (FSRH) has changed over
recent years. The following recommendations are now made for women taken a combined oral
contraceptive (COC) pill containing 30-35 micrograms of ethinylestradiol
If 1 pill is missed (at any time in the cycle)
 take a pill as soon as possible and then continue taking pills daily, one each day
 no additional contraceptive protection needed
If 2 or more pills missed
 take a pill as soon as possible and then continue taking pills daily, one each day
 the women should use condoms or abstain from sex until she has taken pills for 7 days in a
row
 if pills are missed in week 1 (Days 1-7): emergency contraception should be considered if
she had unprotected sexin the pill-free interval or in week 1
 if pills are missed in week 2 (Days 8-14): after seven consecutive days of taking the COC
there is no need for emergency contraception*
 if pills are missed in week 3 (Days 15-21): she should finish the pills in her current pack and
start a new pack the next day; thus omitting the pill free interval
*theoretically women would be protected if they took the COC in a pattern of 7 days on, 7 days off
Q2)_Which one of the following statements regarding the link between intrauterine devices
(IUDs) and ectopic pregnancies is correct?
A. The proportion of pregnancies that are ectopic is increased and the absolute number is
increased
B. The proportion of pregnancies that are ectopic is increased but the absolute number
is decreased
C. Having an intrauterine device has no effect on the rate of ectopic pregnancies
D. The proportion of pregnancies that are ectopic is decreased and the absolute number is
decreased
E. The proportion of pregnancies that are ectopic is decreased but the absolute number is
increased
IUCD - the proportion of pregnancies that are ectopic is increased but the absolute number is
decreased
The October 2011 AKT feedback stated: 'With regard to AKT 13, knowledge about basic
contraceptive issues seemed to be lacking. '
Intrauterine contraceptive devices
Intrauterine contraceptive devices comprise both conventional copper intrauterine devices (IUDs)
and levonorgestrel-releasing intrauterine systems (IUS, Mirena). The IUS is also used in the
management of menorrhagia
Effectiveness
 both the IUD and IUS are more than 99% effective
Mode of action
 IUD: primary mode of action is prevention of fertilisation by causing decreased sperm motility
and survival (possibly an effect of copper ions)
 IUS: levonorgestrel prevents endometrial proliferation and causes cervical mucous
thickening
Counselling
 IUD is effective immediately following insertion
 IUS can be relied upon after 7 days
Potential problems
 IUDs make periods heavier, longer and more painful
 the IUS is associated with initial frequent uterine bleeding and spotting. Later women
typically have intermittent light menses with less dysmenorrhoea and some women become
amenorrhoeic
 uterine perforation: up to 2 per 1000 insertions
 the proportion of pregnancies that are ectopic is increased but the absolute number of
ectopic pregnancies is reduced, compared to a woman not using contraception
 infection: there is a small increased risk of pelvic inflammatory disease in the first 20 days
after insertion but after this period the risk returns to that of a standard population
 expulsion: risk is around 1 in 20, and is most likely to occur in the first 3 months
Q3)_ Whichone of the followingisnot a recognisedadverse effectofthe combinedoral contraceptive
pill?
A. Increasedrisk of ovarian cancer
B. Increasedriskof deepveinthrombosis
C. Increasedriskof breastcancer
D. Increasedriskof ischaemicheartdisease
E. Increasedriskof cervical cancer
Combined oral contraceptive pill
 increased risk of breast and cervical cancer
 protective against ovarian and endometrial cancer
The combined oral contraceptive pill has actually been shown to reduce the risk of ovarian cancer
Combined oral contraceptive pill: advantages/disadvantages
Advantages of combined oral contraceptive pill
 highly effective (failure rate < 1 per 100 woman years)
 doesn't interfere with sex
 contraceptive effects reversible upon stopping
 usually makes periods regular, lighter and less painful
 reduced risk of ovarian, endometrial and colorectal cancer
 may protect against pelvic inflammatory disease
 may reduce ovarian cysts, benign breast disease, acne vulgaris
Disadvantages of combined oral contraceptive pill
 people may forget to take it
 offers no protection against sexually transmitted infections
 increased risk of venous thromboembolic disease
 increased risk of breast and cervical cancer
 increased risk of stroke and ischaemic heart disease (especially in smokers)
 temporary side-effects such as headache, nausea, breast tenderness may be seen
Whilst some users report weight gain whilst taking the combined oral contraceptive pill a Cochrane
review did not support a causal relationship
Q4)_Which one of the following is not an absolute contraindication to combined oral
contraceptive pill use?
A. Blood pressure 165/100 (confirmed on three readings)
B. Continuous use before, during and after a total knee replacement
C. Breast feeding a 10-week-old baby
D. Deep vein thrombosis 9 years ago
E. A 39-year-old who smokes 20 cigarettes/day
Breast feeding < 6 weeks postpartum is UKMEC category 4 where as after this time it is UKMEC
category 3
Combined oral contraceptive pill: contraindications
The decision of whether to start a women on the combined oral contraceptive pill is now guided by
the UK Medical Eligibility Criteria (UKMEC). This scale categorises the potential cautions and
contraindications according to a four point scale, as detailed below:
 UKMEC 1: a condition for which there is no restriction for the use of the contraceptive
method
 UKMEC 2: advantages generally outweigh the disadvantages
 UKMEC 3: disadvantages generally outweigh the advantages
 UKMEC 4: represents an unacceptable health risk
Examples of UKMEC 3 conditions include
 more than 35 years old and smoking less than 15 cigarettes/day
 BMI > 35 kg/m^2*
 migraine without aura and more than 35 years old
 family history of thromboembolic disease in first degree relatives < 45 years
 controlled hypertension
 immobility e.g. wheel chair use
 breast feeding 6 weeks - 6 months postpartum
Examples of UKMEC 4 conditions include
 more than 35 years old and smoking more than 15 cigarettes/day
 migraine with aura
 history of thromboembolic disease or thrombogenic mutation
 history of stroke or ischaemic heart disease
 breast feeding < 6 weeks post-partum
 uncontrolled hypertension
 breast cancer
 major surgery with prolonged immobilisation
Diabetes mellitus diagnosed > 20 years ago is classified as UKMEC 3 or 4 depending on severity
*The UKMEC 4 rating for a BMI > 40 kg/m^2 was removed in 2009.
Q5)_ A 19-year-oldfemale presentsto surgery asking to start an oral contraceptive pill.She has no
significantpast medical history or familyhistory ofnote. If a combinedpill is chosen,what isthe most
appropriate of the options givenbelow?
A. Ethinylestradiol35 mcg withnorethisterone 1mg
B. Ethinylestradiol50 mcg withlevonorgestrel 150 mcg
C. Ethinylestradiol20 mcg withgestodene 75mcg
D. Ethinylestradiol20 mcg withnorethisterone 1mg
E. Ethinylestradiol 30 mcg with levonorgestrel 150 mcg
The faculty recommend a pill with 30 mcg of oestrogen for first-time combined oral contraceptive pill
users
Combined oral contraceptive pill: choice of pill
The combined oral contraceptive method (COC) varies by both the amount of oestrogen and
progestogen and also the presentation (e.g. everyday pill/phasic preparation, patches etc)
For first time users
 consider using a pill containing 30 mcg ethinyloestradiol with levonorgestrel/norethisterone
(e.g. Microgynon 30 - ethinylestradiol 30 mcg with levonorgestrel 150 mcg)
New COC
A product combining 20mcg ethinylestradiol with 3mg drospirenone is soon to be launched in the
UK. In the US and Europe it is branded as Yaz and has an interesting 24/4 regime, as opposed to
the normal 21/7 cycle. The idea is that a shorter pill-free interval is both better for patients with
troublesome premenstrual symptoms and is also more effective at preventing ovulation.
Q6)_A 29-year-old female presents to her GP as she missed her Micronor pill (progestogen
only) this morning and is unsure what to do. She normally takes the pill at around 0830 and it
is now 1100. What advice should be given?
A. Take missed pill now and no further action needed
B. Emergency contraception should be offered
C. Take missed pill now and advise condom use until pill taking re-established for 48 hours
D. Take missed pill now and omit pill break at end of pack
E. Perform a pregnancy test
Progestogen only pill: missed pill
The missed pill rules for the progestogen only pill is as follows:
If < 3 hours* late
 continue as normal
If > 3 hours*
 take missed pill as soon as possible
 continue with rest of pack
 extra precautions (e.g. condoms) should be used until pill taking has been re-established for
48 hours
*for Cerazette (desogestrel) a 12 hour period is allowed
Q7)_A 33-year-old female presents to her GP as she missed her Noriday pill (progestogen
only) this morning and is unsure what to do. She normally takes the pill at around 0900 and it
is now 1230. What advice should be given?
A. Take missed pill as soon as possible and advise condom use until pill taking
re-established for 48 hours
B. Take missed pill as soon as possible and omit pill break at end of pack
C. Perform a pregnancy test
D. Take missed pill as soon as possible and no further action needed
E. Emergency contraception should be offered
The October 2011 AKT feedback stated: 'With regard to AKT 13, knowledge about basic
contraceptive issues seemed to be lacking. '
Progestogen only pill: missed pill
The missed pill rules for the progestogen only pill is as follows:
If < 3 hours* late
 continue as normal
If > 3 hours*
 take missed pill as soon as possible
 continue with rest of pack
 extra precautions (e.g. condoms) should be used until pill taking has been re-established for
48 hours
*for Cerazette (desogestrel) a 12 hour period is allowed
Q8)_A 19-year-old female is prescribed a 7 day course of penicillin for tonsillitis. She is
currently taking Microgynon 30. What is the most appropriate advice regarding
contraception?
A. Use condoms for 7 days only if antibiotic course overlaps with pill free interval
B. Use condoms for 14 days
C. There is no need for extra protection
D. Use condoms for 21 days
E. Use condoms for 7 days
The guidelines have changed. Please see below for more details.
Combined oral contraceptive pill: special situations
Concurrent antibiotic use
 for many years doctors in the UK have advised that the concurrent use of antibiotics may
interfere with the enterohepatic circulation of oestrogen and thus make the combined oral
contraceptive pill ineffective - 'extra- precautions' were advised for the duration of antibiotic
treatment and for 7 days afterwards
 no such precautions are taken in the US or the majority of mainland Europe
 in 2011 the Faculty of Sexual & Reproductive Healthcare produced new guidelines
abandoning this approach. The latest edition of the BNF has been updated in line with this
guidance
 precautions should still be taken with enzyme inducing antibiotics such as rifampicin
Switching combined oral contraceptive pills
 the BNF and Faculty of Sexual & Reproductive Healthcare (FSRH) appear to give
contradictory advice. The Clinical Effectiveness Unit of the FSRH have stated in the
Combined Oral Contraception guidelines that the pill free interval does not need to be
omitted (please see link). The BNF however advises missing the pill free interval if the
progesterone changes. Given the uncertainty it is best to follow the BNF
Q9)_A 19-year-oldfemale isprescribeda 7 day course of amoxicillinfora lowerrespiratory tract
infection.She is currentlytaking Cerazette (desogestrel).Whatisthe most appropriate advice
regarding contraception?
A. Use condomsfor14 days
B. Use condomsfor21 days
C. Use condomsfor7 days
D. There is no needfor extra protections
E. Use condomsfor7 days,onlyantibioticcourse overlapswithpill free interval
Progestogen only pill + antibiotics - no need for extra precautions
Progestogen only pill: counselling
Women who are considering taking the progestogen only pill (POP) should be counselled in a
number of areas:
Potential adverse effects
 irregular vaginal bleeding is the most common problem
Starting the POP
 if commenced up to and including day 5 of the cycle it provides immediate protection,
otherwise additional contraceptive methods (e.g. Condoms) should be used for the first 2
days
 if switching from a combined oral contraceptive (COC) gives immediate protection if
continued directly from the end of a pill packet (i.e. Day 21)
Taking the POP
 should be taken at same time everyday, without a pill free break (unlike the COC)
Missed pills
 if < 3 hours* late: continue as normal
 if > 3 hours*: take missed pill as soon as possible, continue with rest of pack, extra
precautions (e.g. Condoms) should be used until pill taking has been re-established for 48
hours
Other potential problems
 diarrhoea and vomiting: continue taking POP but assume pills have been missed - see
above
 antibiotics: have no effect on the POP**
 liver enzyme inducers may reduce effectiveness
Other information
 discussion on STIs
*for Cerazette (desogestrel) a 12 hour period is allowed
**unless the antibiotic alters the P450 enzyme system, for example rifampicin
Q10)_What is the failure rate of male sterilisation?
A. 1 in 100
B. 1 in 200
C. 1 in 300
D. 1 in 400
E. 1 in 2,000
Male sterilisation - failure rate = 1 in 2,000
Sterilisation
Male sterilisation - vasectomy
 failure rate: 1 per 2,000*
 simple operation, can be done under LA (some GA), go home after a couple of hours
 doesn't work immediately
 semen analysis needs to be performed twice following a vasectomy before a man can have
unprotected sex (usually at 16 and 20 weeks)
 complications: bruising, haematoma, infection, sperm granuloma, chronic testicular pain
(affects between 5-30% men)
 the success rate of vasectomy reversal is up to 55%, if done within 10 years, and
approximately 25% after more than 10 years
Female sterilisation
 failure rate: 1 per 200*
 usually done by laparoscopy under general anaesthetic
 generally done as a day case
 many different techniques involving clips (e.g. Filshie clips) , blockage, rings (Falope rings)
and salpingectomy
 complications: increased risk of ectopic if sterilisation fails, general complications of
GA/laparoscopy
 the current success rate of female sterilisation reversal is between 50-60%
*source = Royal College of Obstetricians and Gynaecologists
Q11)_A 34-year-old female has a TT380 Slimline intrauterine device fitted for contraception on
day 14 of her cycle. She has not been sexually active since her last period. How long will it
take before it can be relied upon as a method of contraception?
A. Immediately
B. 2 days
C. 5 days
D. 7 days
E. Until first day of next period
Contraceptives - time until effective (if not first day period):
 instant: IUD
 2 days: POP
 7 days: COC, injection, implant, IUS
Intrauterine contraceptive devices
Intrauterine contraceptive devices comprise both conventional copper intrauterine devices (IUDs)
and levonorgestrel-releasing intrauterine systems (IUS, Mirena). The IUS is also used in the
management of menorrhagia
Effectiveness
 both the IUD and IUS are more than 99% effective
Mode of action
 IUD: primary mode of action is prevention of fertilisation by causing decreased sperm motility
and survival (possibly an effect of copper ions)
 IUS: levonorgestrel prevents endometrial proliferation and causes cervical mucous
thickening
Counselling
 IUD is effective immediately following insertion
 IUS can be relied upon after 7 days
Potential problems
 IUDs make periods heavier, longer and more painful
 the IUS is associated with initial frequent uterine bleeding and spotting. Later women
typically have intermittent light menses with less dysmenorrhoea and some women become
amenorrhoeic
 uterine perforation: up to 2 per 1000 insertions
 the proportion of pregnancies that are ectopic is increased but the absolute number of
ectopic pregnancies is reduced, compared to a woman not using contraception
 infection: there is a small increased risk of pelvic inflammatory disease in the first 20 days
after insertion but after this period the risk returns to that of a standard population
 expulsion: risk is around 1 in 20, and is most likely to occur in the first 3 months
Q12)_A 19-year-old woman is seen the day after being discharged from hospital following a
termination of pregnancy at 14 weeks. She is keen to start the combined oral contraceptive
(COC) pill despite discussing long acting reversible contraceptives. What is the most
appropriate action?
A. Start COC immediately
B. Start COC after 7 days
C. Refuse to prescribe a contraceptive unless she chooses a long acting reversible
contraceptive
D. Start COC on first day of next period
E. Start COC after 21 days
The COC can be started immediately after a miscarriage or abortion. Women are protected from
pregnancy straight away.
Combined oral contraceptive pill: counselling
Women who are considering taking the combined oral contraceptive pill (COC) should be counselled
in a number of areas:
Potential harms and benefits, including
 the COC is > 99% effective if taken correctly
 small risk of blood clots
 very small risk of heart attacks and strokes
 increased risk of breast cancer and cervical cancer
Advice on taking the pill, including
 if the COC is started within the first 5 days of the cycle then there is no need for additional
contraception. If it is started at any other point in the cycle then alternative contraception
should be used (e.g. condoms) for the first 7 days
 should be taken at the same time everyday
 taken for 21 days then stopped for 7 days - similar uterine bleeding to menstruation
 advice that intercourse during the pill-free period is only safe if the next pack is started on
time
Discussion on situations here efficacy may be reduced*
 if vomiting within 2 hours of taking COC pill
 if taking liver enzyme inducing drugs
Other information
 discussion on STIs
*Concurrent antibiotic use
 for many years doctors in the UK have advised that the concurrent use of antibiotics may
interfere with the enterohepatic circulation of oestrogen and thus make the combined oral
contraceptive pill ineffective - 'extra-precautions' were advised for the duration of antibiotic
treatment and for 7 days afterwards
 no such precautions are taken in the US or the majority of mainland Europe
 in 2011 the Faculty of Sexual & Reproductive Healthcare produced new guidelines
abandoning this approach. The latest edition of the BNF has been updated in line with this
guidance
 precautions should still be taken with enzyme inducing antibiotics such as rifampicin
Q13)_A 44-year-oldfemale has a Mirena(intrauterine system) fittedfor contraceptionon day 12 of
her cycle.How longwill it take before it can be reliedupon as a methodof contraception?
A. Immediately
B. 2 days
C. 5 days
D. 7 days
E. Until firstday of nextperiod
Contraceptives - time until effective (if not first day period):
 instant: IUD
 2 days: POP
 7 days: COC, injection, implant, IUS
Intrauterine contraceptive devices
Intrauterine contraceptive devices comprise both conventional copper intrauterine devices (IUDs)
and levonorgestrel-releasing intrauterine systems (IUS, Mirena). The IUS is also used in the
management of menorrhagia
Effectiveness
 both the IUD and IUS are more than 99% effective
Mode of action
 IUD: primary mode of action is prevention of fertilisation by causing decreased sperm motility
and survival (possibly an effect of copper ions)
 IUS: levonorgestrel prevents endometrial proliferation and causes cervical mucous
thickening
Counselling
 IUD is effective immediately following insertion
 IUS can be relied upon after 7 days
Potential problems
 IUDs make periods heavier, longer and more painful
 the IUS is associated with initial frequent uterine bleeding and spotting. Later women
typically have intermittent light menses with less dysmenorrhoea and some women become
amenorrhoeic
 uterine perforation: up to 2 per 1000 insertions
 the proportion of pregnancies that are ectopic is increased but the absolute number of
ectopic pregnancies is reduced, compared to a woman not using contraception
 infection: there is a small increased risk of pelvic inflammatory disease in the first 20 days
after insertion but after this period the risk returns to that of a standard population
 expulsion: risk is around 1 in 20, and is most likely to occur in the first 3 months
Q14)_ What is the failure rate offemale sterilisation?
A. 1 in100
B. 1 in200
C. 1 in300
D. 1 in400
E. 1 in500
Female sterilisation - failure rate = 1 in 200
Sterilisation
Male sterilisation - vasectomy
 failure rate: 1 per 2,000*
 simple operation, can be done under LA (some GA), go home after a couple of hours
 doesn't work immediately
 semen analysis needs to be performed twice following a vasectomy before a man can have
unprotected sex (usually at 16 and 20 weeks)
 complications: bruising, haematoma, infection, sperm granuloma, chronic testicular pain
(affects between 5-30% men)
 the success rate of vasectomy reversal is up to 55%, if done within 10 years, and
approximately 25% after more than 10 years
Female sterilisation
 failure rate: 1 per 200*
 usually done by laparoscopy under general anaesthetic
 generally done as a day case
 many different techniques involving clips (e.g. Filshie clips) , blockage, rings (Falope rings)
and salpingectomy
 complications: increased risk of ectopic if sterilisation fails, general complications of
GA/laparoscopy
 the current success rate of female sterilisation reversal is between 50-60%
*source = Royal College of Obstetricians and Gynaecologists
Q15)_A 44-year-old man attends for counselling with regards to a vasectomy. Which one of
the following statements is true regarding vasectomy?
A. Vasectomy is effective immediately
B. Female sterilisation is more effective
C. Two negative semen samples should be obtained at 2 and 4 weeks before other
contraceptive methods are stopped
D. Chronic testicular pain is seen in more than 5%of patients
E. Sexual intercourse should be avoided for one month to reduce the chance of a
sperm granuloma
Sterilisation
Male sterilisation - vasectomy
 failure rate: 1 per 2,000*
 simple operation, can be done under LA (some GA), go home after a couple of hours
 doesn't work immediately
 semen analysis needs to be performed twice following a vasectomy before a man can have
unprotected sex (usually at 16 and 20 weeks)
 complications: bruising, haematoma, infection, sperm granuloma, chronic testicular pain
(affects between 5-30% men)
 the success rate of vasectomy reversal is up to 55%, if done within 10 years, and
approximately 25% after more than 10 years
Female sterilisation
 failure rate: 1 per 200*
 usually done by laparoscopy under general anaesthetic
 generally done as a day case
 many different techniques involving clips (e.g. Filshie clips) , blockage, rings (Falope rings)
and salpingectomy
 complications: increased risk of ectopic if sterilisation fails, general complications of
GA/laparoscopy
 the current success rate of female sterilisation reversal is between 50-60%
*source = Royal College of Obstetricians and Gynaecologists
Q16)_ A 33-year-old woman is reviewed following a routine cervical smear. She had an
intrauterine device (IUD) inserted for contraception 2 years ago. She is currently well and
reports no new problems. The smear report shows no evidence of dyskaryosis but states that
Actinomyces-like organisms had been identified. What is the most appropriate management?
A. Remove IUD + high vaginal swab in 1 month
B. No action needed
C. Remove IUD + oral doxycycline
D. Oral metronidazole
E. Remove IUD + oral metronidazole
Actinomyces-like organisms (ALOs) are a commensal of the female genital tract. Current advice
from the Faculty of Sexual and Reproductive Healthcare suggests IUDs need not be removed if the
patient is asymptomatic.
If the patient was symptomatic then IUD removal should be considered along with penicillin therapy
Intrauterine contraceptive devices
Intrauterine contraceptive devices comprise both conventional copper intrauterine devices (IUDs)
and levonorgestrel-releasing intrauterine systems (IUS, Mirena). The IUS is also used in the
management of menorrhagia
Effectiveness
 both the IUD and IUS are more than 99% effective
Mode of action
 IUD: primary mode of action is prevention of fertilisation by causing decreased sperm motility
and survival (possibly an effect of copper ions)
 IUS: levonorgestrel prevents endometrial proliferation and causes cervical mucous
thickening
Counselling
 IUD is effective immediately following insertion
 IUS can be relied upon after 7 days
Potential problems
 IUDs make periods heavier, longer and more painful
 the IUS is associated with initial frequent uterine bleeding and spotting. Later women
typically have intermittent light menses with less dysmenorrhoea and some women become
amenorrhoeic
 uterine perforation: up to 2 per 1000 insertions
 the proportion of pregnancies that are ectopic is increased but the absolute number of
ectopic pregnancies is reduced, compared to a woman not using contraception
 infection: there is a small increased risk of pelvic inflammatory disease in the first 20 days
after insertion but after this period the risk returns to that of a standard population
 expulsion: risk is around 1 in 20, and is most likely to occur in the first 3 months
Q17)_ A female patientasks for advice about havingan intrauterine device inserted(aTT380
Slimline).Whatadvice shouldbe givenregardingthe likelyeffectonher periods?
A. Periodswill tend to be longer,heavierand more painful
B. Theywill stopafter6 monthsin > 50% of users
C. Periodswill tendtobe lighter,shorterandlesspainful
D. Continual, lightbleedingisseenin50%
E. Theywill stopafter6 monthsin > 90% of users
Intrauterine contraceptive devices
Intrauterine contraceptive devices comprise both conventional copper intrauterine devices (IUDs)
and levonorgestrel-releasing intrauterine systems (IUS, Mirena). The IUS is also used in the
management of menorrhagia
Effectiveness
 both the IUD and IUS are more than 99% effective
Mode of action
 IUD: primary mode of action is prevention of fertilisation by causing decreased sperm motility
and survival (possibly an effect of copper ions)
 IUS: levonorgestrel prevents endometrial proliferation and causes cervical mucous
thickening
Counselling
 IUD is effective immediately following insertion
 IUS can be relied upon after 7 days
Potential problems
 IUDs make periods heavier, longer and more painful
 the IUS is associated with initial frequent uterine bleeding and spotting. Later women
typically have intermittent light menses with less dysmenorrhoea and some women become
amenorrhoeic
 uterine perforation: up to 2 per 1000 insertions
 the proportion of pregnancies that are ectopic is increased but the absolute number of
ectopic pregnancies is reduced, compared to a woman not using contraception
 infection: there is a small increased risk of pelvic inflammatory disease in the first 20 days
after insertion but after this period the risk returns to that of a standard population
 expulsion: risk is around 1 in 20, and is most likely to occur in the first 3 months
Q18)_ Whichone of the followingisan absolute contraindicationto combinedoral contraceptive pill
use?
A. Controlledhypertension
B. Historyof cholestasis
C. 36-year-oldwoman smoking20 cigarettes/day
D. BMI of 38 kg/m^2
E. Migraine withoutaura
Combined oral contraceptive pill: contraindications
The decision of whether to start a women on the combined oral contraceptive pill is now guided by
the UK Medical Eligibility Criteria (UKMEC). This scale categorises the potential cautions and
contraindications according to a four point scale, as detailed below:
 UKMEC 1: a condition for which there is no restriction for the use of the contraceptive
method
 UKMEC 2: advantages generally outweigh the disadvantages
 UKMEC 3: disadvantages generally outweigh the advantages
 UKMEC 4: represents an unacceptable health risk
Examples of UKMEC 3 conditions include
 more than 35 years old and smoking less than 15 cigarettes/day
 BMI > 35 kg/m^2*
 migraine without aura and more than 35 years old
 family history of thromboembolic disease in first degree relatives < 45 years
 controlled hypertension
 immobility e.g. wheel chair use
 breast feeding 6 weeks - 6 months postpartum
Examples of UKMEC 4 conditions include
 more than 35 years old and smoking more than 15 cigarettes/day
 migraine with aura
 history of thromboembolic disease or thrombogenic mutation
 history of stroke or ischaemic heart disease
 breast feeding < 6 weeks post-partum
 uncontrolled hypertension
 breast cancer
 major surgery with prolonged immobilisation
Diabetes mellitus diagnosed > 20 years ago is classified as UKMEC 3 or 4 depending on severity
*The UKMEC 4 rating for a BMI > 40 kg/m^2 was removed in 2009.
Q19)_A 22-year-old woman presents for her Depo-provera injection. She apologises as she
forgot about her appointment last week. You calculate she received her last injection 12
weeks and 4 day ago. What is the most appropriate course of action?
A. Do a pregnancy test today + give injection if negative
B. Give injection today and no further action
C. Give injection today + use condoms for 7 days + pregnancy test in 21 days
D. Do not give injection + do pregnancy test in 21 days
E. Give injection today + use condoms for 7 days
Depo-provera can be given up to 14 weeks with no extra precautions
See below - there is a discrepancy between the widely followed 14 week rule and the advice in the
BNF. However, this answer is still consistent with the BNF guidance.
Injectable contraceptives
Depo Provera is the main injectable contraceptive used in the UK*. It contains medroxyprogesterone
acetate 150mg. It is given via in intramuscular injection every 12 weeks. It can however be given up
to 14 weeks after the last dose without the need for extra precautions**
The main method of action is by inhibiting ovulation. Secondary effects include cervical mucus
thickening and endometrial thinning.
Disadvantages include the fact that the injection cannot be reversed once given. There is also a
potential delayed return to fertility (maybe up to 12 months)
Adverse effects
 irregular bleeding
 weight gain
 may potentially increased risk of osteoporosis: should only be used in adolescents if no other
method of contraception is suitable
 not quickly reversible and fertility may return after a varying time
*Noristerat, the other injectable contraceptive licensed in the UK, is rarely used in clinical practice. It
is given every 8 weeks
**the BNF gives different advice, stating a pregnancy test should be done if the interval is greater
than 12 weeks and 5 days - this is however not commonly adhered to in the family planning
community
Q20)_A 25-year-old female presents to her GP as she has missed two consecutive
Microgynon 30 pills. She has taken the Microgynon for the past 5 years and is currently 11
days into a packet of pills. Last night she had sexualintercourse with a new partner but
unfortunately the condom split. What is the correct management?
A. Perform a pregnancy test
B. No action needed
C. Advise condom use for next 7 days
D. Emergency contraception should be offered
E. Omit pill break at end of pack
Updated guidance from the FSRH states the following after a woman has missed two pills:
'If you have missed two or more pills (i.e. more than 48 hours late), anywhere in the pack … continue
taking the rest of the pack as usual and use an extra method of contraception for the next 7 days'
For further information please consult the link to the FSRH guidelines. As this was a new partner
consideration should be given to STI screening after an appropriate interval.
Combined oral contraceptive pill: missed pill
The advice from the Faculty of Sexual and Reproductive Healthcare (FSRH) has changed over
recent years. The following recommendations are now made for women taken a combined oral
contraceptive (COC) pill containing 30-35 micrograms of ethinylestradiol
If 1 pill is missed (at any time in the cycle)
 take a pill as soon as possible and then continue taking pills daily, one each day
 no additional contraceptive protection needed
If 2 or more pills missed
 take a pill as soon as possible and then continue taking pills daily, one each day
 the women should use condoms or abstain from sex until she has taken pills for 7 days in a
row
 if pills are missed in week 1 (Days 1-7): emergency contraception should be considered if
she had unprotected sexin the pill-free interval or in week 1
 if pills are missed in week 2 (Days 8-14): after seven consecutive days of taking the COC
there is no need for emergency contraception*
 if pills are missed in week 3 (Days 15-21): she should finish the pills in her current pack and
start a new pack the next day; thus omitting the pill free interval
*theoretically women would be protected if they took the COC in a pattern of 7 days on, 7 days off
Q21)_Which one of the following is most associated with combined oral contraceptive pill use?
A. Increased dysmenorrhoea
B. Increased incidence of benign breast disease
C. Worsening of acne
D. Increased risk of colorectal cancer
E. Increased risk of cervicalcancer
Combined oral contraceptive pill
 increased risk of breast and cervical cancer
 protective against ovarian and endometrial cancer
Combined oral contraceptive pill: advantages/disadvantages
Advantages of combined oral contraceptive pill
 highly effective (failure rate < 1 per 100 woman years)
 doesn't interfere with sex
 contraceptive effects reversible upon stopping
 usually makes periods regular, lighter and less painful
 reduced risk of ovarian, endometrial and colorectal cancer
 may protect against pelvic inflammatory disease
 may reduce ovarian cysts, benign breast disease, acne vulgaris
Disadvantages of combined oral contraceptive pill
 people may forget to take it
 offers no protection against sexually transmitted infections
 increased risk of venous thromboembolic disease
 increased risk of breast and cervical cancer
 increased risk of stroke and ischaemic heart disease (especially in smokers)
 temporary side-effects such as headache, nausea, breast tenderness may be seen
Whilst some users report weight gain whilst taking the combined oral contraceptive pill a Cochrane
review did not support a causal relationship
Q22)_A 36-year-old female starts Cerazette (desogestrel) on day 7 of her cycle. How long will
it take before it can be relied upon as a method of contraception?
A. Immediately
B. 2 days
C. 5 days
D. 7 days
E. Until first day of next period
Contraceptives - time until effective (if not first day period):
 instant: IUD
 2 days: POP
 7 days: COC, injection, implant, IUS
Progestogen only pill: counselling
Women who are considering taking the progestogen only pill (POP) should be counselled in a
number of areas:
Potential adverse effects
 irregular vaginal bleeding is the most common problem
Starting the POP
 if commenced up to and including day 5 of the cycle it provides immediate protection,
otherwise additional contraceptive methods (e.g. Condoms) should be used for the first 2
days
 if switching from a combined oral contraceptive (COC) gives immediate protection if
continued directly from the end of a pill packet (i.e. Day 21)
Taking the POP
 should be taken at same time everyday, without a pill free break (unlike the COC)
Missed pills
 if < 3 hours* late: continue as normal
 if > 3 hours*: take missed pill as soon as possible, continue with rest of pack, extra
precautions (e.g. Condoms) should be used until pill taking has been re-established for 48
hours
Other potential problems
 diarrhoea and vomiting: continue taking POP but assume pills have been missed - see
above
 antibiotics: have no effect on the POP**
 liver enzyme inducers may reduce effectiveness
Other information
 discussion on STIs
*for Cerazette (desogestrel) a 12 hour period is allowed
**unless the antibiotic alters the P450 enzyme system, for example rifampicin
Q23)_A 29-year-old woman who is 2 weeks postpartum consults you regarding
contraception. She is interested in having an intrauterine device (IUD) inserted and asks
when it could be fitted. She had a emergency caesarean section for failure to progress. What
is the most appropriate advice to give?
A. An IUD can be inserted 4 weeks postpartum
B. An IUD can be inserted 12 months postpartum
C. An IUD can be inserted today
D. An IUD can be inserted 12 weeks postpartum
E. An IUD is contraindicated in the long-term
Guidelines do not suggest there is a need to wait any longer despite the caesarean section.
Intrauterine contraceptive devices: insertion
Very few contraindications to insertion of an intrauterine contraceptive device exist. Below are some
conditions mentioned by the Faculty of Family Planning and Reproductive Health Care. Please see
the link for the full list.
UKMEC Category 3 (Risks outweigh benefits)*
 between 48 hours and 4 weeks postpartum (increased risk of perforation)
 initiation of method** in women with ovarian cancer
UKMEC Category 4 (Unacceptable risk)
 pregnancy
 current pelvic infection, puerperal sepsis, immediate post-septic abortion
 unexplained vaginal bleeding which is suspicious
 uterine fibroids or uterine anatomical abnormalities distorting the uterine cavity
NICE produced guidelines in 2005 on screening for sexually transmitted infections (STI) before
insertion of an intrauterine contraceptive device
 Chlamydia trachomatis in women at risk of STIs
 Neisseria gonorrhoeae in women at risk of STIs, in areas where it is prevalent
 any STIs in women who request it
For women at increased risk of STIs prophylactic antibiotics should be given before inserting an
intrauterine contraceptive device if testing has not yet been completed
*current venous thromboembolism (on anticoagulants) has recently been downgraded from UKMEC
3 to UKMEC 1
**as opposed to continuation of the method
Q24)-A 23-year-old female presents as she would like to switch from Microgynon 30 to
another combined oral contraceptive (COC) due to mood swings.It is decided to start
Marvelon. What advice should be given about switching her COC?
A. 'Overlap' for 7 days after starting the new COC
B. Stop Microgynon, wait until first day of next normal menstrual period before commencing
new pill
C. Finish the current pill packet and the start the new COC without a pill free interval and use
condoms for 7 days
D. Finish the current pill packet and the start the new COC without a pill free interval
E. Switch at any time as the oestrogen content of the pill is the same
Difficult one as the BNF and Faculty of Sexual & Reproductive Healthcare (FSRH) appear to give
contradictory advice. The Clinical Effectiveness Unit of the FSRH have produced a statement that
the pill free interval does not need to be omitted (please see link). The BNF however advises
missing the pill free interval if the progesterone changes. Given the uncertainty it is best to follow the
BNF
Combined oral contraceptive pill: special situations
Concurrent antibiotic use
 for many years doctors in the UK have advised that the concurrent use of antibiotics may
interfere with the enterohepatic circulation of oestrogen and thus make the combined oral
contraceptive pill ineffective - 'extra- precautions' were advised for the duration of antibiotic
treatment and for 7 days afterwards
 no such precautions are taken in the US or the majority of mainland Europe
 in 2011 the Faculty of Sexual & Reproductive Healthcare produced new guidelines
abandoning this approach. The latest edition of the BNF has been updated in line with this
guidance
 precautions should still be taken with enzyme inducing antibiotics such as rifampicin
Switching combined oral contraceptive pills
 the BNF and Faculty of Sexual & Reproductive Healthcare (FSRH) appear to give
contradictory advice. The Clinical Effectiveness Unit of the FSRH have stated in the
Combined Oral Contraception guidelines that the pill free interval does not need to be
omitted (please see link). The BNF however advises missing the pill free interval if the
progesterone changes. Given the uncertainty it is best to follow the BNF
Q25)_ A 27-year-old female presents to her GP as she missed her Cerazette pill (progestogen
only) this morning and is unsure what to do. She normally takes the pill at around 0900 and it
is now 1430. What advice should be given?
A. Emergency contraception should be offered
B. Perform a pregnancy test
C. Take missed pill as soon as possible and omit pill break at end of pack
D. Take missed pill now and no further action needed
E. Take missed pill now and advise condom use until pill taking re-established for 48
hours
As Cerazette has a 12-hour window this patient should take the pill now with no further action being
needed
Progestogen only pill: missed pill
The missed pill rules for the progestogen only pill is as follows:
If < 3 hours* late
 continue as normal
If > 3 hours*
 take missed pill as soon as possible
 continue with rest of pack
 extra precautions (e.g. condoms) should be used until pill taking has been re-established for
48 hours
*for Cerazette (desogestrel) a 12 hour period is allowed
Q26)- What is the most common adverse effect experienced by women taking the
progestogen only pill?
A. Irregular vaginal bleeding
B. Acne
C. Mood swings
D. Reduced libido
E. Weight gain
The October 2011 AKT feedback stated: 'With regard to AKT 13, knowledge about basic
contraceptive issues seemed to be lacking. '
Progestogen only pill: advantages/disadvantages
Advantages
 highly effective (failure rate = 1 per 100 woman years)
 doesn't interfere with sex
 contraceptive effects reversible upon stopping
 can be used whilst breast-feeding
 can be used in situations where the combined oral contraceptive pill is contraindicated e.g. in
smokers > 35 years of age and women with a history of venous thromboembolic disease
Disadvantages
 irregular periods: some users may not have periods whilst others may have irregular or light
periods. This is the most common adverse effect
 doesn't protect against sexually transmitted infections
 increased incidence of functional ovarian cysts
 common side-effects include breast tenderness, weight gain, acne and headaches. These
symptoms generally subside after the first few months
Q27)- A 45-year-old woman requests insertion of an intrauterine device (IUD). Which one of
the following statements regarding the expulsion rate is correct?
A. Occurs in around 1 in 200 women, and is more likely in the first 3 months
B. Occurs in around 1 in 20 women, and is more likely after having the IUD for more
than 3 years
C. Occurs in around 1 in 500 women, and is more likely in the first 3 months
D. Occurs in around 1 in 20 women, and is more likely in the first 3 months
E. Occurs in around 1 in 200 women, and is more likely after having the IUD for more
than 3 years
Expulsion is the most common reason for IUD failure, hence the importance of checking the threads
after each period
Intrauterine contraceptive devices
Intrauterine contraceptive devices comprise both conventional copper intrauterine devices (IUDs)
and levonorgestrel-releasing intrauterine systems (IUS, Mirena). The IUS is also used in the
management of menorrhagia
Effectiveness
 both the IUD and IUS are more than 99% effective
Mode of action
 IUD: primary mode of action is prevention of fertilisation by causing decreased sperm motility
and survival (possibly an effect of copper ions)
 IUS: levonorgestrel prevents endometrial proliferation and causes cervical mucous
thickening
Counselling
 IUD is effective immediately following insertion
 IUS can be relied upon after 7 days
Potential problems
 IUDs make periods heavier, longer and more painful
 the IUS is associated with initial frequent uterine bleeding and spotting. Later women
typically have intermittent light menses with less dysmenorrhoea and some women become
amenorrhoeic
 uterine perforation: up to 2 per 1000 insertions
 the proportion of pregnancies that are ectopic is increased but the absolute number of
ectopic pregnancies is reduced, compared to a woman not using contraception
 infection: there is a small increased risk of pelvic inflammatory disease in the first 20 days
after insertion but after this period the risk returns to that of a standard population
 expulsion: risk is around 1 in 20, and is most likely to occur in the first 3 months
Q28)- Which one of the following contraceptives may decrease bone mineral density in
women?
A. Depo Provera (injectable contraceptive)
B. Progestogen only pill
C. Mirena (intrauterine system)
D. Implanon (implantable contraceptive)
E. Combined oral contraceptive pill
Injectable contraceptives
Depo Provera is the main injectable contraceptive used in the UK*. It contains medroxyprogesterone
acetate 150mg. It is given via in intramuscular injection every 12 weeks. It can however be given up
to 14 weeks after the last dose without the need for extra precautions**
The main method of action is by inhibiting ovulation. Secondary effects include cervical mucus
thickening and endometrial thinning.
Disadvantages include the fact that the injection cannot be reversed once given. There is also a
potential delayed return to fertility (maybe up to 12 months)
Adverse effects
 irregular bleeding
 weight gain
 may potentially increased risk of osteoporosis: should only be used in adolescents if no other
method of contraception is suitable
 not quickly reversible and fertility may return after a varying time
*Noristerat, the other injectable contraceptive licensed in the UK, is rarely used in clinical practice. It
is given every 8 weeks
**the BNF gives different advice, stating a pregnancy test should be done if the interval is greater
than 12 weeks and 5 days - this is however not commonly adhered to in the family planning
community
Q29)_ Which one of the following is less common in women who take the combined oral
contraceptive pill?
A. Stroke
B. Endometrial cancer
C. Pulmonary embolism
D. Cervical cancer
E. Ischaemic heart disease
Combined oral contraceptive pill
 increased risk of breast and cervical cancer
 protective against ovarian and endometrial cancer
Combined oral contraceptive pill: advantages/disadvantages
Advantages of combined oral contraceptive pill
 highly effective (failure rate < 1 per 100 woman years)
 doesn't interfere with sex
 contraceptive effects reversible upon stopping
 usually makes periods regular, lighter and less painful
 reduced risk of ovarian, endometrial and colorectal cancer
 may protect against pelvic inflammatory disease
 may reduce ovarian cysts, benign breast disease, acne vulgaris
Disadvantages of combined oral contraceptive pill
 people may forget to take it
 offers no protection against sexually transmitted infections
 increased risk of venous thromboembolic disease
 increased risk of breast and cervical cancer
 increased risk of stroke and ischaemic heart disease (especially in smokers)
 temporary side-effects such as headache, nausea, breast tenderness may be seen
Whilst some users report weight gain whilst taking the combined oral contraceptive pill a Cochrane
review did not support a causal relationship
Q29)_ Which one of the following statements regarding the link between intrauterine devices
(IUDs) and pelvic inflammatory disease (PID) is correct?
A. Decreased risk in first 20 days then returns to normal
B. There is no link between IUDs and PID
C. Overall decreased risk throughout lifetime of IUD
D. Overall increased risk throughout lifetime of IUD
E. Increased risk in first 20 days then returns to normal
The October 2011 AKT feedback stated: 'With regard to AKT 13, knowledge about basic
contraceptive issues seemed to be lacking. '
Intrauterine contraceptive devices
Intrauterine contraceptive devices comprise both conventional copper intrauterine devices (IUDs)
and levonorgestrel-releasing intrauterine systems (IUS, Mirena). The IUS is also used in the
management of menorrhagia
Effectiveness
 both the IUD and IUS are more than 99% effective
Mode of action
 IUD: primary mode of action is prevention of fertilisation by causing decreased sperm motility
and survival (possibly an effect of copper ions)
 IUS: levonorgestrel prevents endometrial proliferation and causes cervical mucous
thickening
Counselling
 IUD is effective immediately following insertion
 IUS can be relied upon after 7 days
Potential problems
 IUDs make periods heavier, longer and more painful
 the IUS is associated with initial frequent uterine bleeding and spotting. Later women
typically have intermittent light menses with less dysmenorrhoea and some women become
amenorrhoeic
 uterine perforation: up to 2 per 1000 insertions
 the proportion of pregnancies that are ectopic is increased but the absolute number of
ectopic pregnancies is reduced, compared to a woman not using contraception
 infection: there is a small increased risk of pelvic inflammatory disease in the first 20 days
after insertion but after this period the risk returns to that of a standard population
 expulsion: risk is around 1 in 20, and is most likely to occur in the first 3 months
Q30)_ Which one of the following is an absolute contraindication to the use of the
progesterone only pill?
A. Immobility following surgery
B. Breast cancer 3 years ago
C. Previous stroke
D. History of antiphospholipid syndrome
E. Concurrent use of rifampicin
Progestogen only pill: contraindications
The decision of whether to start a women a particular type of contraceptive is now guided by the UK
Medical Eligibility Criteria (UKMEC). This scale categorises the potential cautions and
contraindications according to a four point scale, as detailed below:
 UKMEC 1: a condition for which there is no restriction for the use of the contraceptive
method
 UKMEC 2: advantages generally outweigh the disadvantages
 UKMEC 3: disadvantages generally outweigh the advantages
 UKMEC 4: represents an unacceptable health risk
Examples of UKMEC 3 conditions include
 active liver disease or past tumour
 liver enzyme inducers
 breast cancer more than 5 years ago
 undiagnosed vaginal bleeding
Examples of UKMEC 4 conditions include
 pregnancy
 breast cancer within the last 5 years
Q31)_ You are considering prescribing ulipristal (EllaOne) for a woman who has presented
requesting emergency contraception. How long after unprotected sexual intercourse may
ulipristal be used?
A. 72 hours
B. 96 hours
C. 120 hours (5 days)
D. 144 hours (6 days)
E. 168 hours (7 days)
Ulipristal (EllaOne) - a new type of emergency hormonal contraception, can be used up to 120 hours
post UPSI
Emergency contraception
There are two methods currently available in the UK:
Emergency hormonal contraception
There are now two methods of emergency hormonal contraception ('emergency pill', 'morning-after
pill'); levonorgestrel and ulipristal, a progesterone receptor modulator.
Levonorgestrel
 should be taken as soon as possible - efficacy decreases with time
 must be taken within 72 hrs of unprotected sexual intercourse (UPSI)*
 single dose of levonorgestrel 1.5mg (a progesterone)
 mode of action not fully understood - acts both to stop ovulation and inhibit implantation
 84% effective is used within 72 hours of UPSI
 levonorgestrel is safe and well tolerated. Disturbance of the current menstrual cycle is seen
in a significant minority of women. Vomiting occurs in around 1%
 if vomiting occurs within 2 hours then the dose should be repeated
 can be used more than once in a menstrual cycle if clinically indicated
Ulipristal
 a progesterone receptor modulator currently marketed as EllaOne. The primary
mode of action is thought to be inhibition of ovulation
 30mg oral dose taken as soon as possible, no later than 120 hours after intercourse
 concomitant use with levonorgestrel is not recommended
 may reduce the effectiveness of combined oral contraceptive pills and progesterone only pills
 caution should be exercised in patients with severe asthma
 repeated dosing within the same menstrual cycle is not recommended
Intrauterine device (IUD)
 must be inserted within 5 days of UPSI, or
 if a women presents after more than 5 days then an IUD may be fitted up to 5 days after the
likely ovulation date
 may inhibit fertilisation or implantation
 prophylactic antibiotics may be given if the patient is considered to be at high-risk of sexually
transmitted infection
 is 99% effective regardless of where it is used in the cycle
 may be left in-situ to provide long-term contraception. If the client wishes for the IUD to be
removed it should be at least kept in until the next period
*may be offered after this period as long as the client is aware of reduced effectiveness and
unlicensed indication
Q32)_ A 23-year-old female has a Nexplanon inserted on day 18 of her 28 day cycle. At what
point can the Nexplanon be relied upon to provide contraception?
A. After 2 days
B. At the end of the next menstrual period
C. Immediately
D. After 7 days
E. At the start of the next menstrual period
Contraceptives - time until effective (if not first day period):
 instant: IUD
 2 days: POP
 7 days: COC, injection, implant, IUS
The October 2011 AKT feedback stated: 'With regard to AKT 13, knowledge about basic
contraceptive issues seemed to be lacking. '
Implantable contraceptives
Implanon is a non-biodegradable subdermal contraceptive implant which is currently being phased
out and replaced by Nexplanon. From a pharmacological perspective Nexplanon is the same as
Implanon. The two main differences are:
 the applicator has been redesigned to try and prevent 'deep' insertions (i.e.
subcutaneous/intramuscular)
 it is radiopaque and therefore easier to locate if impalpable
Both versions slowly releases the progestogen hormone etonogestrel. They are typically inserted in
the proximal non-dominant arm, just overlying the tricep. The main mechanism of action is
preventing ovulation. They also work by thickening the cervical mucus.
Key points
 highly effective: failure rate 0.07/100 women/year
 long-acting: lasts 3 years
 doesn't contain oestrogen so can be used if past history of thromboembolism, migraine etc
 can be inserted immediately following a termination of pregnancy
Disadvantages include
 the need for a trained professional to insert and remove device
 additional contraceptive methods are needed for the first 7 days if not inserted on day 1 to 5
of a woman's menstrual cycle
Adverse effects
 irregular/heavy bleeding is the main problem
 'progestogen effects': headache, nausea, breast pain
Q33)_ Which one of the following is an absolute contraindication to combined oral
contraceptive pill use?
A. Concurrent use of St John's Wort
B. Family history of thromboembolic disease in first degree relatives < 45 years
C. Immobility (e.g. wheelchair use)
D. Migraine with aura
E. Diabetes mellitus (diagnosed 11 years ago)
Combined oral contraceptive pill: contraindications
The decision of whether to start a women on the combined oral contraceptive pill is now guided by
the UK Medical Eligibility Criteria (UKMEC). This scale categorises the potential cautions and
contraindications according to a four point scale, as detailed below:
 UKMEC 1: a condition for which there is no restriction for the use of the contraceptive
method
 UKMEC 2: advantages generally outweigh the disadvantages
 UKMEC 3: disadvantages generally outweigh the advantages
 UKMEC 4: represents an unacceptable health risk
Examples of UKMEC 3 conditions include
 more than 35 years old and smoking less than 15 cigarettes/day
 BMI > 35 kg/m^2*
 migraine without aura and more than 35 years old
 family history of thromboembolic disease in first degree relatives < 45 years
 controlled hypertension
 immobility e.g. wheel chair use
 breast feeding 6 weeks - 6 months postpartum
Examples of UKMEC 4 conditions include
 more than 35 years old and smoking more than 15 cigarettes/day
 migraine with aura
 history of thromboembolic disease or thrombogenic mutation
 history of stroke or ischaemic heart disease
 breast feeding < 6 weeks post-partum
 uncontrolled hypertension
 breast cancer
 major surgery with prolonged immobilisation
Diabetes mellitus diagnosed > 20 years ago is classified as UKMEC 3 or 4 depending on severity
*The UKMEC 4 rating for a BMI > 40 kg/m^2 was removed in 2009.
Q34)_ Which one of the following is an absolute contraindication to combined oral
contraceptive pill use?
A. 37-year-old woman smoking 10 cigarettes/day
B. 4 weeks post-partum and breast feeding
C. Being a wheelchair user
D. BMI of 43 kg / m^2
E. Family history of thromboembolic disease in first degree relatives < 45 years
Option A, C, D & E are relative contraindications (UKMEC 3) whilst option B is an absolute
contraindication (UKMEC 4).
The UKMEC 4 rating for a BMI > 40 kg/m^2 was removed in 2009.
Combined oral contraceptive pill: contraindications
The decision of whether to start a women on the combined oral contraceptive pill is now guided by
the UK Medical Eligibility Criteria (UKMEC). This scale categorises the potential cautions and
contraindications according to a four point scale, as detailed below:
 UKMEC 1: a condition for which there is no restriction for the use of the contraceptive
method
 UKMEC 2: advantages generally outweigh the disadvantages
 UKMEC 3: disadvantages generally outweigh the advantages
 UKMEC 4: represents an unacceptable health risk
Examples of UKMEC 3 conditions include
 more than 35 years old and smoking less than 15 cigarettes/day
 BMI > 35 kg/m^2*
 migraine without aura and more than 35 years old
 family history of thromboembolic disease in first degree relatives < 45 years
 controlled hypertension
 immobility e.g. wheel chair use
 breast feeding 6 weeks - 6 months postpartum
Examples of UKMEC 4 conditions include
 more than 35 years old and smoking more than 15 cigarettes/day
 migraine with aura
 history of thromboembolic disease or thrombogenic mutation
 history of stroke or ischaemic heart disease
 breast feeding < 6 weeks post-partum
 uncontrolled hypertension
 breast cancer
 major surgery with prolonged immobilisation
Diabetes mellitus diagnosed > 20 years ago is classified as UKMEC 3 or 4 depending on severity
*The UKMEC 4 rating for a BMI > 40 kg/m^2 was removed in 2009.
Q35)_ What is the main mechanism of action of Cerazette (desogestrel)?
A. Thickens cervical mucous
B. Inhibits ovulation
C. Causes endometrial thinning
D. Inhibits implantation
E. Toxic to sperm
Cerazette inhibits ovulation in around 97-99% of cycles.
Progestogen only pill: types
Second generation
 norethisterone
 levonorgestrel
 ethynodiol diacetate
Third generation
 desogestrel (Cerazette)
Cerazette
 new third generation type of progestogen only pill (POP) containing desogestrel
 inhibits ovulation in the majority of women
 users can take the pill up to 12 hours late rather than 3 hours like other POPs
Q36)_ Each one of the following would decrease the effectiveness of the combined oral
contraceptive pill, except:
A. Rifampicin
B. Sodium valproate
C. Carbamazepine
D. St John's Wort
E. Phenytoin
P450 enzyme system
Induction usually requires prolonged exposure to the inducing drug, as opposed to P450 inhibitors,
where effects are often seen rapidly
Inducers of the P450 system include
 antiepileptics: phenytoin, carbamazepine
 barbiturates: phenobarbitone
 rifampicin
 St John's Wort
 chronic alcohol intake
 griseofulvin
 smoking (affects CYP1A2, reason why smokers require more aminophylline)
Inhibitors of the P450 system include
 antibiotics: ciprofloxacin, erythromycin
 isoniazid
 cimetidine, omeprazole
 amiodarone
 allopurinol
 imidazoles: ketoconazole, fluconazole
 SSRIs: fluoxetine, sertraline
 ritonavir
 sodium valproate
 acute alcohol intake
 quinupristin
Q37)- What is the main mechanism of action of Nexplanon (etonogestrelcontraceptive
implant)?
A. Thickens cervical mucus
B. Thins endometrial lining
C. Inhibition of ovulation
D. Causes fallopian tube dysfunction
E. Prevents implantation
Nexplanon - main mechanism of action is inhibition of ovulation
Implantable contraceptives
Implanon is a non-biodegradable subdermal contraceptive implant which is currently being phased
out and replaced by Nexplanon. From a pharmacological perspective Nexplanon is the same as
Implanon. The two main differences are:
 the applicator has been redesigned to try and prevent 'deep' insertions (i.e.
subcutaneous/intramuscular)
 it is radiopaque and therefore easier to locate if impalpable
Both versions slowly releases the progestogen hormone etonogestrel. They are typically inserted in
the proximal non-dominant arm, just overlying the tricep. The main mechanism of action is
preventing ovulation. They also work by thickening the cervical mucus.
Key points
 highly effective: failure rate 0.07/100 women/year
 long-acting: lasts 3 years
 doesn't contain oestrogen so can be used if past history of thromboembolism, migraine etc
 can be inserted immediately following a termination of pregnancy
Disadvantages include
 the need for a trained professional to insert and remove device
 additional contraceptive methods are needed for the first 7 days if not inserted on day 1 to 5
of a woman's menstrual cycle
Adverse effects
 irregular/heavy bleeding is the main problem
 'progestogen effects': headache, nausea, breast pain
Q38)_ A 16-year-old female with a history of acne requests to start a combined oral
contraceptive pill (COC). She has been taking oxytetracycline for the past 2 months. What is
the most appropriate advice?
A. A double-dose COC should be used
B. Condoms should be used for the first 14 days of COC use
C. She can start using a COC with usual advice
D. A COC is inappropriate and an alternative method of contraception should be
considered
E. Condoms should be used for the first 21 days of COC use
The usual rules regarding starting the COC should still be adhered to, i.e. use condoms for 7 days if
not started on the first day of next period
Combined oral contraceptive pill: special situations
Concurrent antibiotic use
 for many years doctors in the UK have advised that the concurrent use of antibiotics may
interfere with the enterohepatic circulation of oestrogen and thus make the combined oral
contraceptive pill ineffective - 'extra- precautions' were advised for the duration of antibiotic
treatment and for 7 days afterwards
 no such precautions are taken in the US or the majority of mainland Europe
 in 2011 the Faculty of Sexual & Reproductive Healthcare produced new guidelines
abandoning this approach. The latest edition of the BNF has been updated in line with this
guidance
 precautions should still be taken with enzyme inducing antibiotics such as rifampicin
Switching combined oral contraceptive pills
 the BNF and Faculty of Sexual & Reproductive Healthcare (FSRH) appear to give
contradictory advice. The Clinical Effectiveness Unit of the FSRH have stated in the
Combined Oral Contraception guidelines that the pill free interval does not need to be
omitted (please see link). The BNF however advises missing the pill free interval if the
progesterone changes. Given the uncertainty it is best to follow the BNF
Q39)_ A 33-year-old obese woman presents to surgery requesting advice about
contraception. Her body mass index is 36 kg/m^2. What is the most suitable prescription?
A. Desogestrel (Cerazette) 150 mcg od (double dose)
B. Microgynon 30, two tablets a day as directed
C. Microgynon 30, one tablet a day as directed
D. Norethisterone (Noriday) 5mg tds
E. Desogestrel (Cerazette) 75 mcg od (standard dose)
There is little evidence to support the practice of prescribing double the dose of POP in obese
woman. Whilst some practitioners may err on the side of caution and prescribe double the dose of
POP for woman using older preparations this is not justifiable with Cerazette, given the high plasma
levels achieved with this drug.
Progestogen only pill: counselling
Women who are considering taking the progestogen only pill (POP) should be counselled in a
number of areas:
Potential adverse effects
 irregular vaginal bleeding is the most common problem
Starting the POP
 if commenced up to and including day 5 of the cycle it provides immediate protection,
otherwise additional contraceptive methods (e.g. Condoms) should be used for the first 2
days
 if switching from a combined oral contraceptive (COC) gives immediate protection if
continued directly from the end of a pill packet (i.e. Day 21)
Taking the POP
 should be taken at same time everyday, without a pill free break (unlike the COC)
Missed pills
 if < 3 hours* late: continue as normal
 if > 3 hours*: take missed pill as soon as possible, continue with rest of pack, extra
precautions (e.g. Condoms) should be used until pill taking has been re-established for 48
hours
Other potential problems
 diarrhoea and vomiting: continue taking POP but assume pills have been missed - see
above
 antibiotics: have no effect on the POP**
 liver enzyme inducers may reduce effectiveness
Other information
 discussion on STIs
*for Cerazette (desogestrel) a 12 hour period is allowed
**unless the antibiotic alters the P450 enzyme system, for example rifampicin.
Q40)_ A woman rings for advice 4 days post-partum. She is keen to start her progestogen-
only pill again. There have been no problems since giving birth and breast feeding is going
well. What is the most appropriate advice?
A. Contraindicated if breast-feeding
B. Start on day 21, effective immediately
C. Start immediately, effective immediately
D. Start on day 21, effective after 2 days
E. Start on day 7, effective immediately
Post-partum contraception
Progestogen only pill (POP)
 may be started from day 21 - this will provide immediate contraception. If used earlier may
cause breakthrough bleeding
 after day 21 additional contraception should be used for the first 2 days
 a small amount of progestogen enters breast milk but this is not harmful to the infant
Combined oral contraceptive pill (COC)
 absolutely contraindicated - UKMEC 4 - if breast feeding < 6 weeks post-partum
 relatively contraindicated - UKMEC 3 - if breast feeding 6 weeks - 6 months postpartum
 the COC may reduce breast milk production in lactating mothers
 may be started from day 21 - this will provide immediate contraception
 after day 21 additional contraception should be used for the first 7 days
Q41)_ Concurrent use of which one of the following would make combined oral contraceptive
pill less effective?
A. Fluconazole
B. Sodium valproate
C. Allopurinol
D. Isoniazid
E. Carbamazepine
P450 enzyme system
Induction usually requires prolonged exposure to the inducing drug, as opposed to P450 inhibitors,
where effects are often seen rapidly
Inducers of the P450 system include
 antiepileptics: phenytoin, carbamazepine
 barbiturates: phenobarbitone
 rifampicin
 St John's Wort
 chronic alcohol intake
 griseofulvin
 smoking (affects CYP1A2, reason why smokers require more aminophylline)
Inhibitors of the P450 system include
 antibiotics: ciprofloxacin, erythromycin
 isoniazid
 cimetidine, omeprazole
 amiodarone
 allopurinol
 imidazoles: ketoconazole, fluconazole
 SSRIs: fluoxetine, sertraline
 ritonavir
 sodium valproate
 acute alcohol intake
 quinupristin
Q42)_ Concurrent use of which one of the following would make the combined oral
contraceptive pill less effective?
A. Fluconazole
B. Cimetidine
C. St John's Wort
D. Fluoxetine
E. Isoniazid
St John's Wort is an inducers of the P450 enzyme system in the liver. This results in the combined
oral contraceptive pill being metabolised faster and hence may reduce effectiveness.
P450 enzyme system
Induction usually requires prolonged exposure to the inducing drug, as opposed to P450 inhibitors,
where effects are often seen rapidly
Inducers of the P450 system include
 antiepileptics: phenytoin, carbamazepine
 barbiturates: phenobarbitone
 rifampicin
 St John's Wort
 chronic alcohol intake
 griseofulvin
 smoking (affects CYP1A2, reason why smokers require more aminophylline)
Inhibitors of the P450 system include
 antibiotics: ciprofloxacin, erythromycin
 isoniazid
 cimetidine, omeprazole
 amiodarone
 allopurinol
 imidazoles: ketoconazole, fluconazole
 SSRIs: fluoxetine, sertraline
 ritonavir
 sodium valproate
 acute alcohol intake
 quinupristin
Q43)_ A 17-year-old female presents requesting advice as she forgot to take her Microgynon
30 pills on a weekend away. She is normally very good at remembering her pill but has
missed days 10, 11 and 12 of her packet and it is now day 13. Although she took the day 13
pill this morning she is concerned she may become pregnant and she had unprotected
sexual intercourse whilst away. What is the most appropriate management?
A. No action needed
B. No action needed but omit pill break at end of pack
C. Offer emergency contraception - hormonal
D. Offer emergency contraception - intrauterine device
E. No action needed but use condoms for next 7 days
Tough question. As the patient had taken the pill for 7 days in a row previously she is protected for
the next 7 days. The FSRH guidelines state: 'after seven consecutive pills have been taken there is
no need for emergency contraception' - please consult the link. The guidelines also recommend in
this situation using condoms for the next 7 days.
The October 2011 AKT feedback stated: 'With regard to AKT 13, knowledge about basic
contraceptive issues seemed to be lacking. '
Combined oral contraceptive pill: missed pill
The advice from the Faculty of Sexual and Reproductive Healthcare (FSRH) has changed over
recent years. The following recommendations are now made for women taken a combined oral
contraceptive (COC) pill containing 30-35 micrograms of ethinylestradiol
If 1 pill is missed (at any time in the cycle)
 take a pill as soon as possible and then continue taking pills daily, one each day
 no additional contraceptive protection needed
If 2 or more pills missed
 take a pill as soon as possible and then continue taking pills daily, one each day
 the women should use condoms or abstain from sex until she has taken pills for 7 days in a
row
 if pills are missed in week 1 (Days 1-7): emergency contraception should be considered if
she had unprotected sexin the pill-free interval or in week 1
 if pills are missed in week 2 (Days 8-14): after seven consecutive days of taking the COC
there is no need for emergency contraception*
 if pills are missed in week 3 (Days 15-21): she should finish the pills in her current pack and
start a new pack the next day; thus omitting the pill free interval
*theoretically women would be protected if they took the COC in a pattern of 7 days on, 7 days off
Q44)_ Which one of the following is an absolute contraindication to combined oral
contraceptive pill use?
A. Known thrombogenic mutation
B. Family history of thromboembolic disease in first degree relatives < 45 years
C. Hyperlipidaemia
D. Concurrent use of rifampicin
E. Immobility (e.g. wheelchair use).
Combined oral contraceptive pill: contraindications
The decision of whether to start a women on the combined oral contraceptive pill is now guided by
the UK Medical Eligibility Criteria (UKMEC). This scale categorises the potential cautions and
contraindications according to a four point scale, as detailed below:
 UKMEC 1: a condition for which there is no restriction for the use of the contraceptive
method
 UKMEC 2: advantages generally outweigh the disadvantages
 UKMEC 3: disadvantages generally outweigh the advantages
 UKMEC 4: represents an unacceptable health risk
Examples of UKMEC 3 conditions include
 more than 35 years old and smoking less than 15 cigarettes/day
 BMI > 35 kg/m^2*
 migraine without aura and more than 35 years old
 family history of thromboembolic disease in first degree relatives < 45 years
 controlled hypertension
 immobility e.g. wheel chair use
 breast feeding 6 weeks - 6 months postpartum
Examples of UKMEC 4 conditions include
 more than 35 years old and smoking more than 15 cigarettes/day
 migraine with aura
 history of thromboembolic disease or thrombogenic mutation
 history of stroke or ischaemic heart disease
 breast feeding < 6 weeks post-partum
 uncontrolled hypertension
 breast cancer
 major surgery with prolonged immobilisation
Diabetes mellitus diagnosed > 20 years ago is classified as UKMEC 3 or 4 depending on severity
*The UKMEC 4 rating for a BMI > 40 kg/m^2 was removed in 2009.
Q45)_ Which one of the following statements regarding the use of ulipristal (EllaOne) for
emergency contraception is true?
A. Should be used concomitantly with levonorgestrel to increase efficacy
B. Can be used more than once in a menstrual cycle if clinically indicated
C. May reduce the effectiveness of combined oral contraceptive pills
D. Is more effective than an intrauterine device if used before 72 hours has elapsed
E. Two doses are taken 12 hours apart
Emergency contraception
There are two methods currently available in the UK:
Emergency hormonal contraception
There are now two methods of emergency hormonal contraception ('emergency pill', 'morning-after
pill'); levonorgestrel and ulipristal, a progesterone receptor modulator.
Levonorgestrel
 should be taken as soon as possible - efficacy decreases with time
 must be taken within 72 hrs of unprotected sexual intercourse (UPSI)*
 single dose of levonorgestrel 1.5mg (a progesterone)
 mode of action not fully understood - acts both to stop ovulation and inhibit implantation
 84% effective is used within 72 hours of UPSI
 levonorgestrel is safe and well tolerated. Disturbance of the current menstrual cycle is seen
in a significant minority of women. Vomiting occurs in around 1%
 if vomiting occurs within 2 hours then the dose should be repeated
 can be used more than once in a menstrual cycle if clinically indicated
Ulipristal
 a progesterone receptor modulator currently marketed as EllaOne. The primary
mode of action is thought to be inhibition of ovulation
 30mg oral dose taken as soon as possible, no later than 120 hours after intercourse
 concomitant use with levonorgestrel is not recommended
 may reduce the effectiveness of combined oral contraceptive pills and progesterone only pills
 caution should be exercised in patients with severe asthma
 repeated dosing within the same menstrual cycle is not recommended
Intrauterine device (IUD)
 must be inserted within 5 days of UPSI, or
 if a women presents after more than 5 days then an IUD may be fitted up to 5 days after the
likely ovulation date
 may inhibit fertilisation or implantation
 prophylactic antibiotics may be given if the patient is considered to be at high-risk of sexually
transmitted infection
 is 99% effective regardless of where it is used in the cycle
 may be left in-situ to provide long-term contraception. If the client wishes for the IUD to be
removed it should be at least kept in until the next period
*may be offered after this period as long as the client is aware of reduced effectiveness and
unlicensed indication
Q46)_A 19-year-old woman elects to have an implantable contraceptive. What is the main
advantage of Nexplanon over Implanon?
A. Length of contraceptive effect increased from 3 to 5 years
B. Works immediately regardless of when inserted
C. Less irregular bleeding
D. No local anaesthetic required for insertion
E. New design makes the insertion of implants that are too deep less likely
Implantable contraceptives
Implanon is a non-biodegradable subdermal contraceptive implant which is currently being phased
out and replaced by Nexplanon. From a pharmacological perspective Nexplanon is the same as
Implanon. The two main differences are:
 the applicator has been redesigned to try and prevent 'deep' insertions (i.e.
subcutaneous/intramuscular)
 it is radiopaque and therefore easier to locate if impalpable
Both versions slowly releases the progestogen hormone etonogestrel. They are typically inserted in
the proximal non-dominant arm, just overlying the tricep. The main mechanism of action is
preventing ovulation. They also work by thickening the cervical mucus.
Key points
 highly effective: failure rate 0.07/100 women/year
 long-acting: lasts 3 years
 doesn't contain oestrogen so can be used if past history of thromboembolism, migraine etc
 can be inserted immediately following a termination of pregnancy
Disadvantages include
 the need for a trained professional to insert and remove device
 additional contraceptive methods are needed for the first 7 days if not inserted on day 1 to 5
of a woman's menstrual cycle
Adverse effects
 irregular/heavy bleeding is the main problem
 'progestogen effects': headache, nausea, breast pain
Q47)_ You are considering offering one of your female patients the intrauterine system
(Mirena) as a method of contraception. Which one of the following is most likely to represent
a contraindication to this?
A. Epilepsy
B. Being 5 weeks post-partum
C. Current treatment for ovarian cancer
D. Past history of ectopic pregnancy
E. Past history of pelvic inflammatory disease
Intrauterine contraceptive devices: insertion
Very few contraindications to insertion of an intrauterine contraceptive device exist. Below are some
conditions mentioned by the Faculty of Family Planning and Reproductive Health Care. Please see
the link for the full list.
UKMEC Category 3 (Risks outweigh benefits)*
 between 48 hours and 4 weeks postpartum (increased risk of perforation)
 initiation of method** in women with ovarian cancer
UKMEC Category 4 (Unacceptable risk)
 pregnancy
 current pelvic infection, puerperal sepsis, immediate post-septic abortion
 unexplained vaginal bleeding which is suspicious
 uterine fibroids or uterine anatomical abnormalities distorting the uterine cavity
NICE produced guidelines in 2005 on screening for sexually transmitted infections (STI) before
insertion of an intrauterine contraceptive device
 Chlamydia trachomatis in women at risk of STIs
 Neisseria gonorrhoeae in women at risk of STIs, in areas where it is prevalent
 any STIs in women who request it
For women at increased risk of STIs prophylactic antibiotics should be given before inserting an
intrauterine contraceptive device if testing has not yet been completed
*current venous thromboembolism (on anticoagulants) has recently been downgraded from UKMEC
3 to UKMEC 1
**as opposed to continuation of the method
Womens Health ( MRCGP Qs ) : Gynecology.
Q1-A 17-year-old girl presents due to painful periods. These have been present for the past
three years and are associated with a normal amount of blood loss. Her periods are regular
and there is no abnormal bleeding. She is not yet sexually active. What is the most
appropriate first-line treatment?
A. Tranexamic acid
B. Referral for relaxation therapy
C. Paracetamol
D. Combined oral contraceptive pill
E. Ibuprofen
NSAIDs are offered first-line as they will inhibit prostaglandin synthesis, one of the main causes of
dysmenorrhoea pains.
Dysmenorrhoea
Dysmenorrhoea is characterised by excessive pain during the menstrual period. It is traditionally
divided into primary and secondary dysmenorrhoea.
Primary dysmenorrhoea
In primary dysmenorrhoea there is no underlying pelvic pathology. It affects up to 50% of
menstruating women and usually appears within 1-2 years of the menarche. Excessive endometrial
prostaglandin production is thought to be partially responsible.
Features
 pain typically starts just before or within a few hours of the period starting
 suprapubic cramping pains which may radiate to the back or down the thigh
Management
 NSAIDs such as mefenamic acid and ibuprofen are effective in up to 80% of women. They
work by inhibiting prostaglandin production
 combined oral contraceptive pills are used second line
Secondary dysmenorrhoea
Secondary dysmenorrhoea typically develops many years after the menarche and is the result of an
underlying pathology. In contrast to primary dysmenorrhoea the pain usually starts 3-4 days before
the onset of the period. Causes include:
 endometriosis
 adenomyosis
 pelvic inflammatory disease
 intrauterine devices*
 fibroids
Clinical Knowledge Summaries recommend referring all patients with secondary dysmenorrhoea to
gynaecology for investigation.
*this refers to normal copper coils. Note that the intrauterine system (Mirena) may help
dysmenorrhoea
Q2)_ A 57-year-old female presents due to problems with urine leakage over the past six
months. She describes frequent voiding and not always being able to get to the toilet in time.
She denies losing urine when coughing or sneezing.What is the most appropriate initial
treatment?
A. Trial of oxybutynin
B. Bladder retraining
C. Regular toileting
D. Pelvic floor muscle training
E. Topical oestrogen cream
Urinary incontinence - first-line treatment:
 urge incontinence: bladder retraining
 stress incontinence: pelvic floor muscle training
Urinary incontinence
Urinary incontinence (UI) is a common problem, affecting around 4-5% of the UK population. It is
more common in elderly females. NICE released guidance on the management of UI in 2006
Causes
 overactive bladder (OAB)/urge incontinence: due to detrusor over activity
 stress incontinence: leaking small amounts when coughing or laughing
 mixed incontinence: both urge and stress
 overflow incontinence: due to bladder outlet obstruction, e.g. due to prostate enlargement
Initial investigation
 bladder diaries should be completed for a minimum of 3 days
 urine dipstick and culture
Management depends on whether urge or stress UI is the predominant picture. If urge incontinence
is predominant:
 bladder retraining (lasts for a minimum of 6 weeks, the idea is to gradually increase the
intervals between voiding)
 bladder stabilising drugs: immediate release oxybutynin is first-line
 surgical management: e.g. sacral nerve stimulation
If stress incontinence is predominant:
 pelvic floor muscle training (for a minimum of 3 months)
 surgical procedures: e.g. retropubic mid-urethral tape procedures
Q3)_ A 49-year-old female consults her GP asking about hormone replacement therapy
(HRT). What is the most compelling indication for starting HRT?
A. Prevention of ischaemic heart disease
B. Prevention of osteoporosis
C. Reversal of vaginal atrophy
D. Control of vasomotor symptoms such as flushing
E. Prevention of Alzheimer's disease
Main indication for HRT: control of vasomotor symptoms
Hormone replacement therapy: indications
Hormone replacement therapy (HRT) involves the use of a small dose of oestrogen, combined with
a progestogen (in women with a uterus), to help alleviate menopausal symptoms.
The indications for HRT have changed significantly over the past ten years as the long-term risks
became apparent, primarily as a result of the Women's Health Initiative (WHI) study.
Indications
 vasomotor symptoms such as flushing, insomnia and headaches
 premature menopause: should be continued until the age of 50 years
 osteoporosis: but should only be used as second-line treatment
The main indication is the control of vasomotor symptoms. The other indications such as
reversal of vaginal atrophy and prevention of osteoporosis should be treated with other agents
as first-line therapies
Other benefits include a reduced incidence of colorectal cancer
Q4)_ A 24-year-old woman presents for review complaining of heavy periods. This has
been a problem for a number of years now.She has a 28 day cycle and has heavy
bleeding for 5 days. There is no intermenstrual or post-coital bleeding. General and
gynaecological examination is unremarkable. What is the minimum set of investigations
that this patient should be offered?
A. FBC
B. FBC + ferritin
C. No investigations if the gynaecological examination is normal
D. FBC + pelvic ultrasound
E. FBC + TFT
Menorrhagia - all women should have a FBC
Menorrhagia: management
Menorrhagia was previously defined as total blood loss > 80 ml per menses, but it is obviously
difficult to quantify. The management has therefore shifted towards what the woman considers to be
excessive. Prior to the 1990's many women underwent a hysterectomy to treat heavy periods but
since that time the approach has altered radically. The management of menorrhagia now depends
on whether a women needs contraception.
Investigations
 a full blood count should be performed in all women
 further investigations are based upon the history and examination findings
Does not require contraception
 either mefenamic acid 500 mg tds (particularly if there is dysmenorrhoea as well) or
tranexamic acid 1 g tds. Both are started on the first day of the period
 if no improvement then try other drug whilst awaiting referral
Requires contraception, options include
 intrauterine system (Mirena) should be considered first-line
 combined oral contraceptive pill
 long-acting progestogens
Norethisterone 5 mg tds can be used as a short-term option to rapidly stop heavy menstrual
bleeding.
Q5)_ A 31-year-old woman presents as she has noted an offensive, fishy vaginal
discharge. She describes a grey, watery discharge. What is the most likely diagnosis?
A. Trichomonas vaginalis
B. Candida
C. Chlamydia
D. Bacterial vaginosis
E. Physiological discharge
Vaginal discharge
Vaginal discharge is a common presenting symptom and is not always pathological
Common causes
 physiological
 Candida
 Trichomonas vaginalis
 bacterial vaginosis
Less common causes
 whilst cervical infections such as Chlamydia and Gonorrhoea can cause a vaginal discharge
this is rarely the presenting symptoms
 ectropion
 foreign body
 cervical cancer
Key features of the common causes are listed below
Condition Key features
Candida 'Cottage cheese'discharge
Vulvitis
Itch
Trichomonas vaginalis Offensive, yellow/green, frothy discharge
Vulvovaginitis
Strawberry cervix
Bacterial vaginosis Offensive, thin, white/grey, 'fishy' discharge
Q6)- A 53-year-old woman presents with urgency and frequency. Two weeks ago she
consulted with a colleague as she felt 'dry' during intercourse. She has been treated for
urinary tract infections on multiple occasions in the past but urine culture is always
negative. Her only medication is cyclical hormone replacement therapy. A vaginal
examination is performed which shows no evidence of vaginal atrophy and no masses
are felt. An ultrasound is requested:
Both kidneys, spleen and liver are normal size. Outline of the bladder normal. 3 cm simple
ovarian cyst noted on left ovary. Right ovary and uterus normal
What is the most appropriate next step?
A. Refer for urodynamics
B. Pelvic floor muscle training
C. Trial topical oestrogen
D. Urgent referral to gynaecology
E. Refer for bladder retraining
Any ovarian mass in a post-menopausal woman needs to be investigated.
Ovarian enlargement: management
The initial imaging modality for suspected ovarian cysts/tumours is ultrasound. The report will usually
report that the cyst is either:
 simple: unilocular, more likely to be physiological or benign
 complex: multilocular, more likely to be malignant
Management depends on the age of the patient and whether the patient is symptomatic. It should be
remembered that the diagnosis of ovarian cancer is often delayed due to a vague presentation.
Premenopausal women
 a conservative approach may be taken for younger women (especially if < 35 years) as
malignancy is less common. If the cyst is small (e.g. < 5 cm) and reported as 'simple' then it
is highly likely to be benign. A repeat ultrasound should be arranged for 8-12 weeks and
referral considered if it persists.
Postmenopausal women
 by definition physiological cysts are unlikely
 any postmenopausal woman with an ovarian cyst regardless of nature or size should be
referred to gynaecology for assessment
Q7)_A 34-year-old woman is reviewed in surgery. She complains of a long history of deep
pain during intercourse and painful periods. There is a previous history of lower
abdominal pain and in the past she has been diagnosed with irritable bowel syndrome.
She is upset because she would like to start a family but the pain is putting her off sex.
What is the most suitable management?
A. Trial of combined oral contraceptive pill
B. Discuss benefits of intrauterine system
C. Refer to gynaecology
D. Refer for psychosexual counselling
E. Arrange pelvic ultrasound
The combination of deep dyspareunia and lower abdominal pain make a diagnosis of endometriosis
likely. Initial treatment options such as the combined pill are not an option in a woman trying to
conceive. For a definitive diagnosis the patient should ideally have a laparoscopy. A pelvic
ultrasound is not the investigation of choice in endometriosis and may be normal with mild-moderate
disease
Endometriosis
Endometriosis is a common condition characterised by the growth of ectopic endometrial tissue
outside of the uterine cavity. Up to 10-15% of women have a degree of endometriosis
Clinical features
 chronic pelvic pain
 dysmenorrhoea - pain often starts days before bleeding
 deep dyspareunia
 subfertility
Less common features
 urinary symptoms e.g. dysuria, urgency
 dyschezia (painful bowel movements)
Investigation
 laparoscopy is the gold-standard investigation
 there is little role for investigation in primary care (e.g. ultrasound)- if the symptoms are
significant the patient should be referred for a definitive diagnosis
Management depends on clinical features - there is poor correlation between laparoscopic findings
and severity of symptoms
 NSAIDs and other analgesia for symptomatic relief
 combined oral contraceptive pill
 progestogens e.g. medroxyprogesterone acetate
 gonadotrophin-releasing hormone (GnRH) analogues - said to induce a 'pseudomenopause'
due to the low oestrogen levels
 intrauterine system (Mirena)
 drug therapy unfortunately does not seem to have a significant impact on fertility rates
Surgery
 some treatments such as laparoscopic excision and laser treatment of endometriotic ovarian
cysts may improve fertility
Q8-9-10)_ Theme: Pelvic pain
A. Appendicitis B. Ovarian torsion
C. Urogenital prolapse D. Endometriosis
E. Urinary tract infection F. Ovarian cyst
G. Threatened miscarriage H. Irritable bowel syndrome
I. Pelvic inflammatory disease J. Ectopic pregnancy
For each one of the following scenarios please select the most likely diagnosis:
8. A 24-year-old woman presents with crampy suprapubic pain and light vaginal bleeding. Her
last period was 9 weeks ago. Vaginal examination shows a small amount of blood around the
cervix but is otherwise unremarkable.
Threatened miscarriage
Ectopic pregnancy should also be considered here as a number of women will not have the
typical findings of cervical excitation.
9. A 67-year-old woman presents with a heavy, dragging sensation in the suprapubic region.
She also has frequency and urgency.
Urogenital prolapse
Women who have a urogenital prolapse typically describe a 'bearing down', 'heaviness' or
'dragging' sensation.
10. A 29-year-old woman presents with suprapubic pain, irregular periods, dysuria and pain
during intercourse. There is cervical excitation on examination.
Pelvic inflammatory disease
Cervical excitation is found in both pelvic inflammatory disease and ectopic pregnancy.
Pelvic pain
In women the most common cause of pelvic pain is primary dysmenorrhoea. Some women also
experience transient pain in the middle of their cycle secondary to ovulation (mittelschmerz). The
table below gives characteristic features for other conditions causing pelvic pain:
Usually acute
Ectopic pregnancy A typical historyis a female with a history of 6-8 weeks amenorrhoea who presents with
lower abdominal pain and later develops vaginal bleeding
Shoulder tip pain and cervical excitation may be seen
Urinary tract infection Dysuria and frequency are common butwomen mayexperience suprapubic burning
secondaryto cystitis
Appendicitis Pain initial in the central abdomen before localising to the right iliac fossa
Anorexia is common
Tachycardia, low-grade pyrexia, tenderness in RIF
Rovsing's sign:more pain in RIF than LIF when palpating LIF
Pelvic inflammatory
disease
Pelvic pain, fever, deep dyspareunia,vaginal discharge,dysuria and menstrual irregularities
may occur
Cervical excitation may be found on examination
Ovarian torsion Usuallysudden onsetunilateral lower abdominal pain.Onsetmaycoincide with exercise.
Nausea and vomiting are common
Unilateral,tender adnexal mass on examination
Miscarriage Vaginal bleeding and crampylower abdominal pain following a period ofamenorrhoea
Usually chronic
Endometriosis Chronic pelvic pain
Dysmenorrhoea - pain often starts days before bleeding
Deep dyspareunia
Subfertility
Irritable bowel
syndrome
Extremely common.The most consistentfeatures are abdominal pain,bloating and change
in bowel habit
Features such as lethargy, nausea,backache and bladder symptoms mayalso be present
Ovarian cyst Unilateral dull ache which maybe intermittentor only occur during intercourse.Torsion or
rupture may lead to severe abdominal pain
Large cysts may cause abdominal swelling or pressure effects on the bladder
Urogenital prolapse Seen in older women
Sensation ofpressure,heaviness,'bearing-down'
Urinary symptoms:incontinence,frequency,urgency
Q11)_ A 47-year-old woman presents to surgery seeking advice regarding contraception. She
has recently started in a new relationship but is unsure if she requires contraception, as she
thinks she may be going through the menopause. She is e xperiencing hot flushes and her
last period was 7 months ago. What is the most appropriate advice?
A. Contraception is needed until 36 months after her last period
B. Contraception is needed until 18 months after her last period
C. Contraception is needed until 24 months after her last period
D. Contraception is needed until 12 months after her last period
E. She no longer requires contraception
Need for contraception after the menopause
 12 months after the last period in women > 50 years
 24 months after the last period in women < 50 years
Menopause
The average women in the UK goes through the menopause when she is 51 years old. The
climacteric is the period prior to the menopause where women may experience symptoms, as
ovarian function starts to fail
Diagnosis
 12 months after the last period in women > 50 years
 24 months after the last period in women < 50 years
It is recommend to use effective contraception until the diagnosis has been confirmed using the
above criteria
Q12)_You receive the results of a 29-year-old female who has recently had a routine cervical
smear. Her last smear 4 years ago was reported as normal. The results are reported as
follows:
Moderate dyskaryosis
What is the most appropriate management?
A. Repeat smear in 6 months
B. Repeat smear immediately
C. Refer to a gynaecological oncologist
D. Repeat smear in 3 months
E. Refer to colposcopy
Cervical cancer screening: interpretation of results
The table below outlines the management of abnormal cervical smears (around 5% of all smears).
Cervical intraepithelial neoplasia is abbreviated to CIN
Mild dyskaryosis Consistentwith CIN I. Previouslywomen were offered a repeatsmear after 6 months and
referral for colposcopyifchanges persisted
Whilstthis is still acceptable itis now considered bestpractice to refer women immediatelyfor
colposcopy
Moderate
dyskaryosis
Consistentwith CIN II. Refer for colposcopy
Severe
dyskaryosis
Consistentwith CIN III. Refer for colposcopy
Inadequate Repeatsmear - if persistent(3 inadequate samples),assessmentbycolposcopy
Q13-14-15)-Theme: Ovarian cysts
A. Dermoid cyst (teratoma) B. Endometriotic cyst
C. Granulosa cell tumour D. Clear cell tumour
E. Corpus luteum cyst F. Mucinous cystadenoma
G. Follicular cyst H. Serous cystadenoma
I. Dysgerminoma J. Fibroma
For each one of the following please select the answer from the list above:
13. Most common type of ovarian pathology associated with Meigs' syndrome
The correct answer is Fibroma
Meigs' syndrome is a benign ovarian tumour (usually a fibroma) associated with ascites and pleural
effusion
14. Most common benign ovarian tumour in women under the age of 25 years
The correct answer is Dermoid cyst (teratoma)
15. The most common cause of ovarian enlargement in women of a reproductive age
The correct answer is Follicular cyst
Ovarian cysts: types
Womens Health
Womens Health
Womens Health
Womens Health
Womens Health
Womens Health
Womens Health
Womens Health
Womens Health
Womens Health
Womens Health
Womens Health
Womens Health
Womens Health
Womens Health
Womens Health
Womens Health
Womens Health
Womens Health
Womens Health
Womens Health
Womens Health
Womens Health
Womens Health
Womens Health
Womens Health
Womens Health
Womens Health
Womens Health
Womens Health
Womens Health
Womens Health
Womens Health
Womens Health
Womens Health
Womens Health
Womens Health
Womens Health
Womens Health
Womens Health
Womens Health
Womens Health
Womens Health
Womens Health
Womens Health
Womens Health
Womens Health
Womens Health
Womens Health
Womens Health
Womens Health
Womens Health
Womens Health
Womens Health
Womens Health
Womens Health
Womens Health
Womens Health
Womens Health
Womens Health
Womens Health
Womens Health
Womens Health
Womens Health
Womens Health
Womens Health
Womens Health
Womens Health
Womens Health
Womens Health
Womens Health
Womens Health
Womens Health
Womens Health
Womens Health
Womens Health
Womens Health
Womens Health
Womens Health
Womens Health
Womens Health
Womens Health
Womens Health
Womens Health
Womens Health
Womens Health
Womens Health
Womens Health
Womens Health
Womens Health
Womens Health
Womens Health
Womens Health
Womens Health
Womens Health
Womens Health
Womens Health
Womens Health
Womens Health
Womens Health
Womens Health
Womens Health
Womens Health
Womens Health
Womens Health
Womens Health
Womens Health
Womens Health
Womens Health
Womens Health
Womens Health
Womens Health
Womens Health
Womens Health
Womens Health
Womens Health
Womens Health
Womens Health
Womens Health
Womens Health
Womens Health
Womens Health
Womens Health
Womens Health
Womens Health
Womens Health
Womens Health
Womens Health
Womens Health
Womens Health
Womens Health
Womens Health
Womens Health
Womens Health
Womens Health
Womens Health
Womens Health
Womens Health
Womens Health
Womens Health
Womens Health
Womens Health
Womens Health
Womens Health
Womens Health
Womens Health
Womens Health
Womens Health
Womens Health
Womens Health
Womens Health
Womens Health
Womens Health
Womens Health
Womens Health
Womens Health
Womens Health
Womens Health
Womens Health
Womens Health
Womens Health
Womens Health
Womens Health
Womens Health
Womens Health
Womens Health
Womens Health
Womens Health
Womens Health
Womens Health
Womens Health
Womens Health

More Related Content

What's hot

Evidence -based Management of PCOS
Evidence -based Management of PCOSEvidence -based Management of PCOS
Evidence -based Management of PCOSpogisurabaya
 
PANEL DISCUSSION Management Of Adolescent PCOS And Associated Fertility Conc...
PANEL DISCUSSION Management Of Adolescent PCOS And Associated Fertility Conc...PANEL DISCUSSION Management Of Adolescent PCOS And Associated Fertility Conc...
PANEL DISCUSSION Management Of Adolescent PCOS And Associated Fertility Conc...Lifecare Centre
 
Step by Step Guide to Menopause Hormone Therapy by Dr. laxmi Shrikhande
Step by Step Guide to Menopause Hormone Therapy by Dr. laxmi ShrikhandeStep by Step Guide to Menopause Hormone Therapy by Dr. laxmi Shrikhande
Step by Step Guide to Menopause Hormone Therapy by Dr. laxmi ShrikhandeDr.Laxmi Agrawal Shrikhande
 
Nausea & Vomiting in Pregnancy :an update Dr Sharda Jain
Nausea & Vomiting in Pregnancy :an update Dr Sharda Jain Nausea & Vomiting in Pregnancy :an update Dr Sharda Jain
Nausea & Vomiting in Pregnancy :an update Dr Sharda Jain Lifecare Centre
 
Presentation on Fertility Challenges in Polycystic Ovary Syndrome (PCOS)
Presentation on Fertility Challenges in Polycystic Ovary Syndrome (PCOS)Presentation on Fertility Challenges in Polycystic Ovary Syndrome (PCOS)
Presentation on Fertility Challenges in Polycystic Ovary Syndrome (PCOS)Dr.Laxmi Agrawal Shrikhande
 
Blanca Vazquez, MD
Blanca Vazquez, MDBlanca Vazquez, MD
Blanca Vazquez, MDNYU FACES
 
Tender Love and Care (TLC) in Recurrent Pregnancy Loss (RPL) Dr Sharda Jain D...
Tender Love and Care (TLC) in Recurrent Pregnancy Loss (RPL) Dr Sharda Jain D...Tender Love and Care (TLC) in Recurrent Pregnancy Loss (RPL) Dr Sharda Jain D...
Tender Love and Care (TLC) in Recurrent Pregnancy Loss (RPL) Dr Sharda Jain D...Lifecare Centre
 
Recurrent miscarriage: how to manage
Recurrent miscarriage: how to manageRecurrent miscarriage: how to manage
Recurrent miscarriage: how to manageHesham Al-Inany
 
step by step guide to menopause hormone therapy (MHT)
step by step guide to menopause hormone therapy (MHT)step by step guide to menopause hormone therapy (MHT)
step by step guide to menopause hormone therapy (MHT)Dr.Laxmi Agrawal Shrikhande
 
Overview of Hormones and Epilepsy
Overview of Hormones and EpilepsyOverview of Hormones and Epilepsy
Overview of Hormones and EpilepsyEFEPA
 
Dienogest+ Ethinyl Estradiol Role in oral contraception & Acne Dr Sharda Jain...
Dienogest+ Ethinyl Estradiol Role in oral contraception & Acne Dr Sharda Jain...Dienogest+ Ethinyl Estradiol Role in oral contraception & Acne Dr Sharda Jain...
Dienogest+ Ethinyl Estradiol Role in oral contraception & Acne Dr Sharda Jain...Lifecare Centre
 
Acute Severe Hypertension in pregnancy
Acute Severe Hypertension in pregnancyAcute Severe Hypertension in pregnancy
Acute Severe Hypertension in pregnancyveerendrakumar cm
 
Polikistik Over Sendromu - PCOS - www.jinekolojivegebelik.com
Polikistik Over Sendromu - PCOS - www.jinekolojivegebelik.comPolikistik Over Sendromu - PCOS - www.jinekolojivegebelik.com
Polikistik Over Sendromu - PCOS - www.jinekolojivegebelik.comjinekolojivegebelik.com
 
Women and Epilepsy: Taking Care of Yourself
Women and Epilepsy: Taking Care of YourselfWomen and Epilepsy: Taking Care of Yourself
Women and Epilepsy: Taking Care of YourselfEFEPA
 

What's hot (20)

Evidence -based Management of PCOS
Evidence -based Management of PCOSEvidence -based Management of PCOS
Evidence -based Management of PCOS
 
Premenstrual Syndrome
Premenstrual SyndromePremenstrual Syndrome
Premenstrual Syndrome
 
PANEL DISCUSSION Management Of Adolescent PCOS And Associated Fertility Conc...
PANEL DISCUSSION Management Of Adolescent PCOS And Associated Fertility Conc...PANEL DISCUSSION Management Of Adolescent PCOS And Associated Fertility Conc...
PANEL DISCUSSION Management Of Adolescent PCOS And Associated Fertility Conc...
 
Step by Step Guide to Menopause Hormone Therapy by Dr. laxmi Shrikhande
Step by Step Guide to Menopause Hormone Therapy by Dr. laxmi ShrikhandeStep by Step Guide to Menopause Hormone Therapy by Dr. laxmi Shrikhande
Step by Step Guide to Menopause Hormone Therapy by Dr. laxmi Shrikhande
 
Nausea & Vomiting in Pregnancy :an update Dr Sharda Jain
Nausea & Vomiting in Pregnancy :an update Dr Sharda Jain Nausea & Vomiting in Pregnancy :an update Dr Sharda Jain
Nausea & Vomiting in Pregnancy :an update Dr Sharda Jain
 
Presentation on Fertility Challenges in Polycystic Ovary Syndrome (PCOS)
Presentation on Fertility Challenges in Polycystic Ovary Syndrome (PCOS)Presentation on Fertility Challenges in Polycystic Ovary Syndrome (PCOS)
Presentation on Fertility Challenges in Polycystic Ovary Syndrome (PCOS)
 
Blanca Vazquez, MD
Blanca Vazquez, MDBlanca Vazquez, MD
Blanca Vazquez, MD
 
Non-contraceptive Benefits of COCP
Non-contraceptive Benefits of COCPNon-contraceptive Benefits of COCP
Non-contraceptive Benefits of COCP
 
Tender Love and Care (TLC) in Recurrent Pregnancy Loss (RPL) Dr Sharda Jain D...
Tender Love and Care (TLC) in Recurrent Pregnancy Loss (RPL) Dr Sharda Jain D...Tender Love and Care (TLC) in Recurrent Pregnancy Loss (RPL) Dr Sharda Jain D...
Tender Love and Care (TLC) in Recurrent Pregnancy Loss (RPL) Dr Sharda Jain D...
 
Recurrent miscarriage: how to manage
Recurrent miscarriage: how to manageRecurrent miscarriage: how to manage
Recurrent miscarriage: how to manage
 
step by step guide to menopause hormone therapy (MHT)
step by step guide to menopause hormone therapy (MHT)step by step guide to menopause hormone therapy (MHT)
step by step guide to menopause hormone therapy (MHT)
 
Overview of Hormones and Epilepsy
Overview of Hormones and EpilepsyOverview of Hormones and Epilepsy
Overview of Hormones and Epilepsy
 
Dienogest+ Ethinyl Estradiol Role in oral contraception & Acne Dr Sharda Jain...
Dienogest+ Ethinyl Estradiol Role in oral contraception & Acne Dr Sharda Jain...Dienogest+ Ethinyl Estradiol Role in oral contraception & Acne Dr Sharda Jain...
Dienogest+ Ethinyl Estradiol Role in oral contraception & Acne Dr Sharda Jain...
 
Pms Recent Guidelines
Pms Recent GuidelinesPms Recent Guidelines
Pms Recent Guidelines
 
Acute Severe Hypertension in pregnancy
Acute Severe Hypertension in pregnancyAcute Severe Hypertension in pregnancy
Acute Severe Hypertension in pregnancy
 
Subchorionic haemorrhages
Subchorionic haemorrhagesSubchorionic haemorrhages
Subchorionic haemorrhages
 
Polikistik Over Sendromu - PCOS - www.jinekolojivegebelik.com
Polikistik Over Sendromu - PCOS - www.jinekolojivegebelik.comPolikistik Over Sendromu - PCOS - www.jinekolojivegebelik.com
Polikistik Over Sendromu - PCOS - www.jinekolojivegebelik.com
 
HBH pullman 2016
HBH pullman 2016HBH pullman 2016
HBH pullman 2016
 
Obesity and gynecology
Obesity and gynecologyObesity and gynecology
Obesity and gynecology
 
Women and Epilepsy: Taking Care of Yourself
Women and Epilepsy: Taking Care of YourselfWomen and Epilepsy: Taking Care of Yourself
Women and Epilepsy: Taking Care of Yourself
 

Similar to Womens Health

Contraception_Lecture.ppt
Contraception_Lecture.pptContraception_Lecture.ppt
Contraception_Lecture.pptLathan34
 
Contraception2.ppt
Contraception2.pptContraception2.ppt
Contraception2.pptKutemwa1
 
Barries, OCPs ,EC,and Injectables.pptx
Barries, OCPs ,EC,and Injectables.pptxBarries, OCPs ,EC,and Injectables.pptx
Barries, OCPs ,EC,and Injectables.pptxGetanehLiknaw
 
Safe Prescribing of Second Line Combined Oral Contraceptive
Safe Prescribing of Second Line Combined Oral ContraceptiveSafe Prescribing of Second Line Combined Oral Contraceptive
Safe Prescribing of Second Line Combined Oral Contraceptivemeducationdotnet
 
CME PRESENTATION LSC ONG.pptx
CME PRESENTATION LSC ONG.pptxCME PRESENTATION LSC ONG.pptx
CME PRESENTATION LSC ONG.pptxGabrielLucas70256
 
hormonalcontraception-210101111102 (1).pdf
hormonalcontraception-210101111102 (1).pdfhormonalcontraception-210101111102 (1).pdf
hormonalcontraception-210101111102 (1).pdfAderawAlemie
 
Contraception a long time for w2019.pptx
Contraception a long time for w2019.pptxContraception a long time for w2019.pptx
Contraception a long time for w2019.pptxerikabaizhanova
 
2. Family planing level3.pptx
2. Family planing level3.pptx2. Family planing level3.pptx
2. Family planing level3.pptxnuradinman89
 
2. Family planing level3.pptx
2. Family planing level3.pptx2. Family planing level3.pptx
2. Family planing level3.pptxnuradinman89
 
Contraception
ContraceptionContraception
ContraceptionAlya Imad
 
Family planning
Family planning Family planning
Family planning tariggally
 
Update on contraception
Update on contraceptionUpdate on contraception
Update on contraceptiondrajaysd
 

Similar to Womens Health (20)

post abortion.pptx
post abortion.pptxpost abortion.pptx
post abortion.pptx
 
Family planning
Family planningFamily planning
Family planning
 
Contraception_Lecture.ppt
Contraception_Lecture.pptContraception_Lecture.ppt
Contraception_Lecture.ppt
 
Contraception
ContraceptionContraception
Contraception
 
Contraception2.ppt
Contraception2.pptContraception2.ppt
Contraception2.ppt
 
Barries, OCPs ,EC,and Injectables.pptx
Barries, OCPs ,EC,and Injectables.pptxBarries, OCPs ,EC,and Injectables.pptx
Barries, OCPs ,EC,and Injectables.pptx
 
767 missed pillrules%20
767 missed pillrules%20767 missed pillrules%20
767 missed pillrules%20
 
Safe Prescribing of Second Line Combined Oral Contraceptive
Safe Prescribing of Second Line Combined Oral ContraceptiveSafe Prescribing of Second Line Combined Oral Contraceptive
Safe Prescribing of Second Line Combined Oral Contraceptive
 
CME PRESENTATION LSC ONG.pptx
CME PRESENTATION LSC ONG.pptxCME PRESENTATION LSC ONG.pptx
CME PRESENTATION LSC ONG.pptx
 
hormonalcontraception-210101111102 (1).pdf
hormonalcontraception-210101111102 (1).pdfhormonalcontraception-210101111102 (1).pdf
hormonalcontraception-210101111102 (1).pdf
 
Contraceptive.pptx
Contraceptive.pptxContraceptive.pptx
Contraceptive.pptx
 
contraception qs1pdf.pdf
contraception qs1pdf.pdfcontraception qs1pdf.pdf
contraception qs1pdf.pdf
 
Contraception a long time for w2019.pptx
Contraception a long time for w2019.pptxContraception a long time for w2019.pptx
Contraception a long time for w2019.pptx
 
2. Family planing level3.pptx
2. Family planing level3.pptx2. Family planing level3.pptx
2. Family planing level3.pptx
 
2. Family planing level3.pptx
2. Family planing level3.pptx2. Family planing level3.pptx
2. Family planing level3.pptx
 
Contraception
ContraceptionContraception
Contraception
 
Combined oral contraceptive pills
Combined oral contraceptive pillsCombined oral contraceptive pills
Combined oral contraceptive pills
 
Injectables.ppt
Injectables.pptInjectables.ppt
Injectables.ppt
 
Family planning
Family planning Family planning
Family planning
 
Update on contraception
Update on contraceptionUpdate on contraception
Update on contraception
 

More from ssnsharifa

Evidence Update in UTI
Evidence Update in UTIEvidence Update in UTI
Evidence Update in UTIssnsharifa
 
Drug Interaction
Drug InteractionDrug Interaction
Drug Interactionssnsharifa
 
High Alert Medication
 High Alert Medication  High Alert Medication
High Alert Medication ssnsharifa
 
Case Presentation
Case PresentationCase Presentation
Case Presentationssnsharifa
 
Vaginal Discharge
Vaginal DischargeVaginal Discharge
Vaginal Dischargessnsharifa
 
Mediations during month of ramadhan
Mediations during month of ramadhanMediations during month of ramadhan
Mediations during month of ramadhanssnsharifa
 
Trauma Management in Primary Care Setting
Trauma Management in Primary Care SettingTrauma Management in Primary Care Setting
Trauma Management in Primary Care Settingssnsharifa
 
Occupational asthma
Occupational asthmaOccupational asthma
Occupational asthmassnsharifa
 
Low carbs diet
Low carbs dietLow carbs diet
Low carbs dietssnsharifa
 
Medicine MRCGP Endocrinology.
Medicine MRCGP  Endocrinology.Medicine MRCGP  Endocrinology.
Medicine MRCGP Endocrinology.ssnsharifa
 
Deramatology MRCGP Qs
Deramatology MRCGP QsDeramatology MRCGP Qs
Deramatology MRCGP Qsssnsharifa
 

More from ssnsharifa (19)

Evidence Update in UTI
Evidence Update in UTIEvidence Update in UTI
Evidence Update in UTI
 
Drug Interaction
Drug InteractionDrug Interaction
Drug Interaction
 
High Alert Medication
 High Alert Medication  High Alert Medication
High Alert Medication
 
Case Presentation
Case PresentationCase Presentation
Case Presentation
 
Vaginal Discharge
Vaginal DischargeVaginal Discharge
Vaginal Discharge
 
Mediations during month of ramadhan
Mediations during month of ramadhanMediations during month of ramadhan
Mediations during month of ramadhan
 
Trauma Management in Primary Care Setting
Trauma Management in Primary Care SettingTrauma Management in Primary Care Setting
Trauma Management in Primary Care Setting
 
Short Stature
Short StatureShort Stature
Short Stature
 
Osteoprosis
OsteoprosisOsteoprosis
Osteoprosis
 
Occupational asthma
Occupational asthmaOccupational asthma
Occupational asthma
 
Monoarthritis
MonoarthritisMonoarthritis
Monoarthritis
 
Menopause
MenopauseMenopause
Menopause
 
Low carbs diet
Low carbs dietLow carbs diet
Low carbs diet
 
Insomnia
InsomniaInsomnia
Insomnia
 
Fever & Rash
Fever & RashFever & Rash
Fever & Rash
 
Medicine MRCGP Endocrinology.
Medicine MRCGP  Endocrinology.Medicine MRCGP  Endocrinology.
Medicine MRCGP Endocrinology.
 
ENT MRCGP Qs
ENT MRCGP QsENT MRCGP Qs
ENT MRCGP Qs
 
Deramatology MRCGP Qs
Deramatology MRCGP QsDeramatology MRCGP Qs
Deramatology MRCGP Qs
 
HPV
HPVHPV
HPV
 

Recently uploaded

Kesar Bagh Call Girl Price 9548273370 , Lucknow Call Girls Service
Kesar Bagh Call Girl Price 9548273370 , Lucknow Call Girls ServiceKesar Bagh Call Girl Price 9548273370 , Lucknow Call Girls Service
Kesar Bagh Call Girl Price 9548273370 , Lucknow Call Girls Servicemakika9823
 
Russian Call Girl Brookfield - 7001305949 Escorts Service 50% Off with Cash O...
Russian Call Girl Brookfield - 7001305949 Escorts Service 50% Off with Cash O...Russian Call Girl Brookfield - 7001305949 Escorts Service 50% Off with Cash O...
Russian Call Girl Brookfield - 7001305949 Escorts Service 50% Off with Cash O...narwatsonia7
 
Russian Call Girls in Pune Tanvi 9907093804 Short 1500 Night 6000 Best call g...
Russian Call Girls in Pune Tanvi 9907093804 Short 1500 Night 6000 Best call g...Russian Call Girls in Pune Tanvi 9907093804 Short 1500 Night 6000 Best call g...
Russian Call Girls in Pune Tanvi 9907093804 Short 1500 Night 6000 Best call g...Miss joya
 
Call Girl Bangalore Nandini 7001305949 Independent Escort Service Bangalore
Call Girl Bangalore Nandini 7001305949 Independent Escort Service BangaloreCall Girl Bangalore Nandini 7001305949 Independent Escort Service Bangalore
Call Girl Bangalore Nandini 7001305949 Independent Escort Service Bangalorenarwatsonia7
 
Call Girls Horamavu WhatsApp Number 7001035870 Meeting With Bangalore Escorts
Call Girls Horamavu WhatsApp Number 7001035870 Meeting With Bangalore EscortsCall Girls Horamavu WhatsApp Number 7001035870 Meeting With Bangalore Escorts
Call Girls Horamavu WhatsApp Number 7001035870 Meeting With Bangalore Escortsvidya singh
 
Artifacts in Nuclear Medicine with Identifying and resolving artifacts.
Artifacts in Nuclear Medicine with Identifying and resolving artifacts.Artifacts in Nuclear Medicine with Identifying and resolving artifacts.
Artifacts in Nuclear Medicine with Identifying and resolving artifacts.MiadAlsulami
 
Russian Call Girls in Chennai Pallavi 9907093804 Independent Call Girls Servi...
Russian Call Girls in Chennai Pallavi 9907093804 Independent Call Girls Servi...Russian Call Girls in Chennai Pallavi 9907093804 Independent Call Girls Servi...
Russian Call Girls in Chennai Pallavi 9907093804 Independent Call Girls Servi...Nehru place Escorts
 
Call Girls Service Jaipur Grishma WhatsApp ❤8445551418 VIP Call Girls Jaipur
Call Girls Service Jaipur Grishma WhatsApp ❤8445551418 VIP Call Girls JaipurCall Girls Service Jaipur Grishma WhatsApp ❤8445551418 VIP Call Girls Jaipur
Call Girls Service Jaipur Grishma WhatsApp ❤8445551418 VIP Call Girls Jaipurparulsinha
 
High Profile Call Girls Jaipur Vani 8445551418 Independent Escort Service Jaipur
High Profile Call Girls Jaipur Vani 8445551418 Independent Escort Service JaipurHigh Profile Call Girls Jaipur Vani 8445551418 Independent Escort Service Jaipur
High Profile Call Girls Jaipur Vani 8445551418 Independent Escort Service Jaipurparulsinha
 
Call Girls Service In Shyam Nagar Whatsapp 8445551418 Independent Escort Service
Call Girls Service In Shyam Nagar Whatsapp 8445551418 Independent Escort ServiceCall Girls Service In Shyam Nagar Whatsapp 8445551418 Independent Escort Service
Call Girls Service In Shyam Nagar Whatsapp 8445551418 Independent Escort Serviceparulsinha
 
Call Girls Service Chennai Jiya 7001305949 Independent Escort Service Chennai
Call Girls Service Chennai Jiya 7001305949 Independent Escort Service ChennaiCall Girls Service Chennai Jiya 7001305949 Independent Escort Service Chennai
Call Girls Service Chennai Jiya 7001305949 Independent Escort Service ChennaiNehru place Escorts
 
Hi,Fi Call Girl In Mysore Road - 7001305949 | 24x7 Service Available Near Me
Hi,Fi Call Girl In Mysore Road - 7001305949 | 24x7 Service Available Near MeHi,Fi Call Girl In Mysore Road - 7001305949 | 24x7 Service Available Near Me
Hi,Fi Call Girl In Mysore Road - 7001305949 | 24x7 Service Available Near Menarwatsonia7
 
Call Girls Doddaballapur Road Just Call 7001305949 Top Class Call Girl Servic...
Call Girls Doddaballapur Road Just Call 7001305949 Top Class Call Girl Servic...Call Girls Doddaballapur Road Just Call 7001305949 Top Class Call Girl Servic...
Call Girls Doddaballapur Road Just Call 7001305949 Top Class Call Girl Servic...narwatsonia7
 
Bangalore Call Girls Marathahalli 📞 9907093804 High Profile Service 100% Safe
Bangalore Call Girls Marathahalli 📞 9907093804 High Profile Service 100% SafeBangalore Call Girls Marathahalli 📞 9907093804 High Profile Service 100% Safe
Bangalore Call Girls Marathahalli 📞 9907093804 High Profile Service 100% Safenarwatsonia7
 
Low Rate Call Girls Ambattur Anika 8250192130 Independent Escort Service Amba...
Low Rate Call Girls Ambattur Anika 8250192130 Independent Escort Service Amba...Low Rate Call Girls Ambattur Anika 8250192130 Independent Escort Service Amba...
Low Rate Call Girls Ambattur Anika 8250192130 Independent Escort Service Amba...narwatsonia7
 
Aspirin presentation slides by Dr. Rewas Ali
Aspirin presentation slides by Dr. Rewas AliAspirin presentation slides by Dr. Rewas Ali
Aspirin presentation slides by Dr. Rewas AliRewAs ALI
 
Call Girls Yelahanka Just Call 7001305949 Top Class Call Girl Service Available
Call Girls Yelahanka Just Call 7001305949 Top Class Call Girl Service AvailableCall Girls Yelahanka Just Call 7001305949 Top Class Call Girl Service Available
Call Girls Yelahanka Just Call 7001305949 Top Class Call Girl Service Availablenarwatsonia7
 
Call Girls Service Noida Maya 9711199012 Independent Escort Service Noida
Call Girls Service Noida Maya 9711199012 Independent Escort Service NoidaCall Girls Service Noida Maya 9711199012 Independent Escort Service Noida
Call Girls Service Noida Maya 9711199012 Independent Escort Service NoidaPooja Gupta
 
Bangalore Call Girls Majestic 📞 9907093804 High Profile Service 100% Safe
Bangalore Call Girls Majestic 📞 9907093804 High Profile Service 100% SafeBangalore Call Girls Majestic 📞 9907093804 High Profile Service 100% Safe
Bangalore Call Girls Majestic 📞 9907093804 High Profile Service 100% Safenarwatsonia7
 
Low Rate Call Girls Pune Esha 9907093804 Short 1500 Night 6000 Best call girl...
Low Rate Call Girls Pune Esha 9907093804 Short 1500 Night 6000 Best call girl...Low Rate Call Girls Pune Esha 9907093804 Short 1500 Night 6000 Best call girl...
Low Rate Call Girls Pune Esha 9907093804 Short 1500 Night 6000 Best call girl...Miss joya
 

Recently uploaded (20)

Kesar Bagh Call Girl Price 9548273370 , Lucknow Call Girls Service
Kesar Bagh Call Girl Price 9548273370 , Lucknow Call Girls ServiceKesar Bagh Call Girl Price 9548273370 , Lucknow Call Girls Service
Kesar Bagh Call Girl Price 9548273370 , Lucknow Call Girls Service
 
Russian Call Girl Brookfield - 7001305949 Escorts Service 50% Off with Cash O...
Russian Call Girl Brookfield - 7001305949 Escorts Service 50% Off with Cash O...Russian Call Girl Brookfield - 7001305949 Escorts Service 50% Off with Cash O...
Russian Call Girl Brookfield - 7001305949 Escorts Service 50% Off with Cash O...
 
Russian Call Girls in Pune Tanvi 9907093804 Short 1500 Night 6000 Best call g...
Russian Call Girls in Pune Tanvi 9907093804 Short 1500 Night 6000 Best call g...Russian Call Girls in Pune Tanvi 9907093804 Short 1500 Night 6000 Best call g...
Russian Call Girls in Pune Tanvi 9907093804 Short 1500 Night 6000 Best call g...
 
Call Girl Bangalore Nandini 7001305949 Independent Escort Service Bangalore
Call Girl Bangalore Nandini 7001305949 Independent Escort Service BangaloreCall Girl Bangalore Nandini 7001305949 Independent Escort Service Bangalore
Call Girl Bangalore Nandini 7001305949 Independent Escort Service Bangalore
 
Call Girls Horamavu WhatsApp Number 7001035870 Meeting With Bangalore Escorts
Call Girls Horamavu WhatsApp Number 7001035870 Meeting With Bangalore EscortsCall Girls Horamavu WhatsApp Number 7001035870 Meeting With Bangalore Escorts
Call Girls Horamavu WhatsApp Number 7001035870 Meeting With Bangalore Escorts
 
Artifacts in Nuclear Medicine with Identifying and resolving artifacts.
Artifacts in Nuclear Medicine with Identifying and resolving artifacts.Artifacts in Nuclear Medicine with Identifying and resolving artifacts.
Artifacts in Nuclear Medicine with Identifying and resolving artifacts.
 
Russian Call Girls in Chennai Pallavi 9907093804 Independent Call Girls Servi...
Russian Call Girls in Chennai Pallavi 9907093804 Independent Call Girls Servi...Russian Call Girls in Chennai Pallavi 9907093804 Independent Call Girls Servi...
Russian Call Girls in Chennai Pallavi 9907093804 Independent Call Girls Servi...
 
Call Girls Service Jaipur Grishma WhatsApp ❤8445551418 VIP Call Girls Jaipur
Call Girls Service Jaipur Grishma WhatsApp ❤8445551418 VIP Call Girls JaipurCall Girls Service Jaipur Grishma WhatsApp ❤8445551418 VIP Call Girls Jaipur
Call Girls Service Jaipur Grishma WhatsApp ❤8445551418 VIP Call Girls Jaipur
 
High Profile Call Girls Jaipur Vani 8445551418 Independent Escort Service Jaipur
High Profile Call Girls Jaipur Vani 8445551418 Independent Escort Service JaipurHigh Profile Call Girls Jaipur Vani 8445551418 Independent Escort Service Jaipur
High Profile Call Girls Jaipur Vani 8445551418 Independent Escort Service Jaipur
 
Call Girls Service In Shyam Nagar Whatsapp 8445551418 Independent Escort Service
Call Girls Service In Shyam Nagar Whatsapp 8445551418 Independent Escort ServiceCall Girls Service In Shyam Nagar Whatsapp 8445551418 Independent Escort Service
Call Girls Service In Shyam Nagar Whatsapp 8445551418 Independent Escort Service
 
Call Girls Service Chennai Jiya 7001305949 Independent Escort Service Chennai
Call Girls Service Chennai Jiya 7001305949 Independent Escort Service ChennaiCall Girls Service Chennai Jiya 7001305949 Independent Escort Service Chennai
Call Girls Service Chennai Jiya 7001305949 Independent Escort Service Chennai
 
Hi,Fi Call Girl In Mysore Road - 7001305949 | 24x7 Service Available Near Me
Hi,Fi Call Girl In Mysore Road - 7001305949 | 24x7 Service Available Near MeHi,Fi Call Girl In Mysore Road - 7001305949 | 24x7 Service Available Near Me
Hi,Fi Call Girl In Mysore Road - 7001305949 | 24x7 Service Available Near Me
 
Call Girls Doddaballapur Road Just Call 7001305949 Top Class Call Girl Servic...
Call Girls Doddaballapur Road Just Call 7001305949 Top Class Call Girl Servic...Call Girls Doddaballapur Road Just Call 7001305949 Top Class Call Girl Servic...
Call Girls Doddaballapur Road Just Call 7001305949 Top Class Call Girl Servic...
 
Bangalore Call Girls Marathahalli 📞 9907093804 High Profile Service 100% Safe
Bangalore Call Girls Marathahalli 📞 9907093804 High Profile Service 100% SafeBangalore Call Girls Marathahalli 📞 9907093804 High Profile Service 100% Safe
Bangalore Call Girls Marathahalli 📞 9907093804 High Profile Service 100% Safe
 
Low Rate Call Girls Ambattur Anika 8250192130 Independent Escort Service Amba...
Low Rate Call Girls Ambattur Anika 8250192130 Independent Escort Service Amba...Low Rate Call Girls Ambattur Anika 8250192130 Independent Escort Service Amba...
Low Rate Call Girls Ambattur Anika 8250192130 Independent Escort Service Amba...
 
Aspirin presentation slides by Dr. Rewas Ali
Aspirin presentation slides by Dr. Rewas AliAspirin presentation slides by Dr. Rewas Ali
Aspirin presentation slides by Dr. Rewas Ali
 
Call Girls Yelahanka Just Call 7001305949 Top Class Call Girl Service Available
Call Girls Yelahanka Just Call 7001305949 Top Class Call Girl Service AvailableCall Girls Yelahanka Just Call 7001305949 Top Class Call Girl Service Available
Call Girls Yelahanka Just Call 7001305949 Top Class Call Girl Service Available
 
Call Girls Service Noida Maya 9711199012 Independent Escort Service Noida
Call Girls Service Noida Maya 9711199012 Independent Escort Service NoidaCall Girls Service Noida Maya 9711199012 Independent Escort Service Noida
Call Girls Service Noida Maya 9711199012 Independent Escort Service Noida
 
Bangalore Call Girls Majestic 📞 9907093804 High Profile Service 100% Safe
Bangalore Call Girls Majestic 📞 9907093804 High Profile Service 100% SafeBangalore Call Girls Majestic 📞 9907093804 High Profile Service 100% Safe
Bangalore Call Girls Majestic 📞 9907093804 High Profile Service 100% Safe
 
Low Rate Call Girls Pune Esha 9907093804 Short 1500 Night 6000 Best call girl...
Low Rate Call Girls Pune Esha 9907093804 Short 1500 Night 6000 Best call girl...Low Rate Call Girls Pune Esha 9907093804 Short 1500 Night 6000 Best call girl...
Low Rate Call Girls Pune Esha 9907093804 Short 1500 Night 6000 Best call girl...
 

Womens Health

  • 1. Womens Health ( MRCGP Qs ) : Contraception Q1)_An 18-year-old female presentstoher GP as she has missedone of her Microgynon30 pills yesterdaymorning. She has taken Microgynonfor the past 2 years and is currently 4 days intoa packet of pills.She had sexual intercourse last nightand isunsure what to do. What is the correct management? A. Advise condomuse fornext7 days B. Performa pregnancytest C. Omitpill breakat endof pack D. No action needed E. Emergencycontraceptionshouldbe offered As she has only missed one pill no action is needed. For further information please consult the link to the FSRH guidelines. The October 2011 AKT feedback stated: 'With regard to AKT 13, knowledge about basic contraceptive issues seemed to be lacking. ' Combined oral contraceptive pill: missed pill The advice from the Faculty of Sexual and Reproductive Healthcare (FSRH) has changed over recent years. The following recommendations are now made for women taken a combined oral contraceptive (COC) pill containing 30-35 micrograms of ethinylestradiol If 1 pill is missed (at any time in the cycle)  take a pill as soon as possible and then continue taking pills daily, one each day  no additional contraceptive protection needed If 2 or more pills missed  take a pill as soon as possible and then continue taking pills daily, one each day  the women should use condoms or abstain from sex until she has taken pills for 7 days in a row  if pills are missed in week 1 (Days 1-7): emergency contraception should be considered if she had unprotected sexin the pill-free interval or in week 1  if pills are missed in week 2 (Days 8-14): after seven consecutive days of taking the COC there is no need for emergency contraception*
  • 2.  if pills are missed in week 3 (Days 15-21): she should finish the pills in her current pack and start a new pack the next day; thus omitting the pill free interval *theoretically women would be protected if they took the COC in a pattern of 7 days on, 7 days off Q2)_Which one of the following statements regarding the link between intrauterine devices (IUDs) and ectopic pregnancies is correct? A. The proportion of pregnancies that are ectopic is increased and the absolute number is increased B. The proportion of pregnancies that are ectopic is increased but the absolute number is decreased C. Having an intrauterine device has no effect on the rate of ectopic pregnancies D. The proportion of pregnancies that are ectopic is decreased and the absolute number is decreased E. The proportion of pregnancies that are ectopic is decreased but the absolute number is increased IUCD - the proportion of pregnancies that are ectopic is increased but the absolute number is decreased The October 2011 AKT feedback stated: 'With regard to AKT 13, knowledge about basic contraceptive issues seemed to be lacking. ' Intrauterine contraceptive devices Intrauterine contraceptive devices comprise both conventional copper intrauterine devices (IUDs) and levonorgestrel-releasing intrauterine systems (IUS, Mirena). The IUS is also used in the management of menorrhagia Effectiveness  both the IUD and IUS are more than 99% effective Mode of action  IUD: primary mode of action is prevention of fertilisation by causing decreased sperm motility and survival (possibly an effect of copper ions)
  • 3.  IUS: levonorgestrel prevents endometrial proliferation and causes cervical mucous thickening Counselling  IUD is effective immediately following insertion  IUS can be relied upon after 7 days Potential problems  IUDs make periods heavier, longer and more painful  the IUS is associated with initial frequent uterine bleeding and spotting. Later women typically have intermittent light menses with less dysmenorrhoea and some women become amenorrhoeic  uterine perforation: up to 2 per 1000 insertions  the proportion of pregnancies that are ectopic is increased but the absolute number of ectopic pregnancies is reduced, compared to a woman not using contraception  infection: there is a small increased risk of pelvic inflammatory disease in the first 20 days after insertion but after this period the risk returns to that of a standard population  expulsion: risk is around 1 in 20, and is most likely to occur in the first 3 months Q3)_ Whichone of the followingisnot a recognisedadverse effectofthe combinedoral contraceptive pill? A. Increasedrisk of ovarian cancer B. Increasedriskof deepveinthrombosis C. Increasedriskof breastcancer D. Increasedriskof ischaemicheartdisease E. Increasedriskof cervical cancer Combined oral contraceptive pill  increased risk of breast and cervical cancer
  • 4.  protective against ovarian and endometrial cancer The combined oral contraceptive pill has actually been shown to reduce the risk of ovarian cancer Combined oral contraceptive pill: advantages/disadvantages Advantages of combined oral contraceptive pill  highly effective (failure rate < 1 per 100 woman years)  doesn't interfere with sex  contraceptive effects reversible upon stopping  usually makes periods regular, lighter and less painful  reduced risk of ovarian, endometrial and colorectal cancer  may protect against pelvic inflammatory disease  may reduce ovarian cysts, benign breast disease, acne vulgaris Disadvantages of combined oral contraceptive pill  people may forget to take it  offers no protection against sexually transmitted infections  increased risk of venous thromboembolic disease  increased risk of breast and cervical cancer  increased risk of stroke and ischaemic heart disease (especially in smokers)  temporary side-effects such as headache, nausea, breast tenderness may be seen Whilst some users report weight gain whilst taking the combined oral contraceptive pill a Cochrane review did not support a causal relationship Q4)_Which one of the following is not an absolute contraindication to combined oral contraceptive pill use? A. Blood pressure 165/100 (confirmed on three readings) B. Continuous use before, during and after a total knee replacement C. Breast feeding a 10-week-old baby
  • 5. D. Deep vein thrombosis 9 years ago E. A 39-year-old who smokes 20 cigarettes/day Breast feeding < 6 weeks postpartum is UKMEC category 4 where as after this time it is UKMEC category 3 Combined oral contraceptive pill: contraindications The decision of whether to start a women on the combined oral contraceptive pill is now guided by the UK Medical Eligibility Criteria (UKMEC). This scale categorises the potential cautions and contraindications according to a four point scale, as detailed below:  UKMEC 1: a condition for which there is no restriction for the use of the contraceptive method  UKMEC 2: advantages generally outweigh the disadvantages  UKMEC 3: disadvantages generally outweigh the advantages  UKMEC 4: represents an unacceptable health risk Examples of UKMEC 3 conditions include  more than 35 years old and smoking less than 15 cigarettes/day  BMI > 35 kg/m^2*  migraine without aura and more than 35 years old  family history of thromboembolic disease in first degree relatives < 45 years  controlled hypertension  immobility e.g. wheel chair use  breast feeding 6 weeks - 6 months postpartum Examples of UKMEC 4 conditions include  more than 35 years old and smoking more than 15 cigarettes/day  migraine with aura  history of thromboembolic disease or thrombogenic mutation  history of stroke or ischaemic heart disease  breast feeding < 6 weeks post-partum  uncontrolled hypertension  breast cancer  major surgery with prolonged immobilisation Diabetes mellitus diagnosed > 20 years ago is classified as UKMEC 3 or 4 depending on severity
  • 6. *The UKMEC 4 rating for a BMI > 40 kg/m^2 was removed in 2009. Q5)_ A 19-year-oldfemale presentsto surgery asking to start an oral contraceptive pill.She has no significantpast medical history or familyhistory ofnote. If a combinedpill is chosen,what isthe most appropriate of the options givenbelow? A. Ethinylestradiol35 mcg withnorethisterone 1mg B. Ethinylestradiol50 mcg withlevonorgestrel 150 mcg C. Ethinylestradiol20 mcg withgestodene 75mcg D. Ethinylestradiol20 mcg withnorethisterone 1mg E. Ethinylestradiol 30 mcg with levonorgestrel 150 mcg The faculty recommend a pill with 30 mcg of oestrogen for first-time combined oral contraceptive pill users Combined oral contraceptive pill: choice of pill The combined oral contraceptive method (COC) varies by both the amount of oestrogen and progestogen and also the presentation (e.g. everyday pill/phasic preparation, patches etc) For first time users  consider using a pill containing 30 mcg ethinyloestradiol with levonorgestrel/norethisterone (e.g. Microgynon 30 - ethinylestradiol 30 mcg with levonorgestrel 150 mcg) New COC A product combining 20mcg ethinylestradiol with 3mg drospirenone is soon to be launched in the UK. In the US and Europe it is branded as Yaz and has an interesting 24/4 regime, as opposed to the normal 21/7 cycle. The idea is that a shorter pill-free interval is both better for patients with troublesome premenstrual symptoms and is also more effective at preventing ovulation. Q6)_A 29-year-old female presents to her GP as she missed her Micronor pill (progestogen only) this morning and is unsure what to do. She normally takes the pill at around 0830 and it is now 1100. What advice should be given? A. Take missed pill now and no further action needed B. Emergency contraception should be offered
  • 7. C. Take missed pill now and advise condom use until pill taking re-established for 48 hours D. Take missed pill now and omit pill break at end of pack E. Perform a pregnancy test Progestogen only pill: missed pill The missed pill rules for the progestogen only pill is as follows: If < 3 hours* late  continue as normal If > 3 hours*  take missed pill as soon as possible  continue with rest of pack  extra precautions (e.g. condoms) should be used until pill taking has been re-established for 48 hours *for Cerazette (desogestrel) a 12 hour period is allowed Q7)_A 33-year-old female presents to her GP as she missed her Noriday pill (progestogen only) this morning and is unsure what to do. She normally takes the pill at around 0900 and it is now 1230. What advice should be given? A. Take missed pill as soon as possible and advise condom use until pill taking re-established for 48 hours B. Take missed pill as soon as possible and omit pill break at end of pack C. Perform a pregnancy test D. Take missed pill as soon as possible and no further action needed E. Emergency contraception should be offered The October 2011 AKT feedback stated: 'With regard to AKT 13, knowledge about basic contraceptive issues seemed to be lacking. ' Progestogen only pill: missed pill The missed pill rules for the progestogen only pill is as follows: If < 3 hours* late
  • 8.  continue as normal If > 3 hours*  take missed pill as soon as possible  continue with rest of pack  extra precautions (e.g. condoms) should be used until pill taking has been re-established for 48 hours *for Cerazette (desogestrel) a 12 hour period is allowed Q8)_A 19-year-old female is prescribed a 7 day course of penicillin for tonsillitis. She is currently taking Microgynon 30. What is the most appropriate advice regarding contraception? A. Use condoms for 7 days only if antibiotic course overlaps with pill free interval B. Use condoms for 14 days C. There is no need for extra protection D. Use condoms for 21 days E. Use condoms for 7 days The guidelines have changed. Please see below for more details. Combined oral contraceptive pill: special situations Concurrent antibiotic use  for many years doctors in the UK have advised that the concurrent use of antibiotics may interfere with the enterohepatic circulation of oestrogen and thus make the combined oral contraceptive pill ineffective - 'extra- precautions' were advised for the duration of antibiotic treatment and for 7 days afterwards  no such precautions are taken in the US or the majority of mainland Europe  in 2011 the Faculty of Sexual & Reproductive Healthcare produced new guidelines abandoning this approach. The latest edition of the BNF has been updated in line with this guidance  precautions should still be taken with enzyme inducing antibiotics such as rifampicin Switching combined oral contraceptive pills
  • 9.  the BNF and Faculty of Sexual & Reproductive Healthcare (FSRH) appear to give contradictory advice. The Clinical Effectiveness Unit of the FSRH have stated in the Combined Oral Contraception guidelines that the pill free interval does not need to be omitted (please see link). The BNF however advises missing the pill free interval if the progesterone changes. Given the uncertainty it is best to follow the BNF Q9)_A 19-year-oldfemale isprescribeda 7 day course of amoxicillinfora lowerrespiratory tract infection.She is currentlytaking Cerazette (desogestrel).Whatisthe most appropriate advice regarding contraception? A. Use condomsfor14 days B. Use condomsfor21 days C. Use condomsfor7 days D. There is no needfor extra protections E. Use condomsfor7 days,onlyantibioticcourse overlapswithpill free interval Progestogen only pill + antibiotics - no need for extra precautions Progestogen only pill: counselling Women who are considering taking the progestogen only pill (POP) should be counselled in a number of areas: Potential adverse effects  irregular vaginal bleeding is the most common problem Starting the POP  if commenced up to and including day 5 of the cycle it provides immediate protection, otherwise additional contraceptive methods (e.g. Condoms) should be used for the first 2 days  if switching from a combined oral contraceptive (COC) gives immediate protection if continued directly from the end of a pill packet (i.e. Day 21)
  • 10. Taking the POP  should be taken at same time everyday, without a pill free break (unlike the COC) Missed pills  if < 3 hours* late: continue as normal  if > 3 hours*: take missed pill as soon as possible, continue with rest of pack, extra precautions (e.g. Condoms) should be used until pill taking has been re-established for 48 hours Other potential problems  diarrhoea and vomiting: continue taking POP but assume pills have been missed - see above  antibiotics: have no effect on the POP**  liver enzyme inducers may reduce effectiveness Other information  discussion on STIs *for Cerazette (desogestrel) a 12 hour period is allowed **unless the antibiotic alters the P450 enzyme system, for example rifampicin Q10)_What is the failure rate of male sterilisation? A. 1 in 100 B. 1 in 200 C. 1 in 300 D. 1 in 400 E. 1 in 2,000
  • 11. Male sterilisation - failure rate = 1 in 2,000 Sterilisation Male sterilisation - vasectomy  failure rate: 1 per 2,000*  simple operation, can be done under LA (some GA), go home after a couple of hours  doesn't work immediately  semen analysis needs to be performed twice following a vasectomy before a man can have unprotected sex (usually at 16 and 20 weeks)  complications: bruising, haematoma, infection, sperm granuloma, chronic testicular pain (affects between 5-30% men)  the success rate of vasectomy reversal is up to 55%, if done within 10 years, and approximately 25% after more than 10 years Female sterilisation  failure rate: 1 per 200*  usually done by laparoscopy under general anaesthetic  generally done as a day case  many different techniques involving clips (e.g. Filshie clips) , blockage, rings (Falope rings) and salpingectomy  complications: increased risk of ectopic if sterilisation fails, general complications of GA/laparoscopy  the current success rate of female sterilisation reversal is between 50-60% *source = Royal College of Obstetricians and Gynaecologists Q11)_A 34-year-old female has a TT380 Slimline intrauterine device fitted for contraception on day 14 of her cycle. She has not been sexually active since her last period. How long will it take before it can be relied upon as a method of contraception? A. Immediately
  • 12. B. 2 days C. 5 days D. 7 days E. Until first day of next period Contraceptives - time until effective (if not first day period):  instant: IUD  2 days: POP  7 days: COC, injection, implant, IUS Intrauterine contraceptive devices Intrauterine contraceptive devices comprise both conventional copper intrauterine devices (IUDs) and levonorgestrel-releasing intrauterine systems (IUS, Mirena). The IUS is also used in the management of menorrhagia Effectiveness  both the IUD and IUS are more than 99% effective Mode of action  IUD: primary mode of action is prevention of fertilisation by causing decreased sperm motility and survival (possibly an effect of copper ions)  IUS: levonorgestrel prevents endometrial proliferation and causes cervical mucous thickening Counselling  IUD is effective immediately following insertion  IUS can be relied upon after 7 days Potential problems  IUDs make periods heavier, longer and more painful
  • 13.  the IUS is associated with initial frequent uterine bleeding and spotting. Later women typically have intermittent light menses with less dysmenorrhoea and some women become amenorrhoeic  uterine perforation: up to 2 per 1000 insertions  the proportion of pregnancies that are ectopic is increased but the absolute number of ectopic pregnancies is reduced, compared to a woman not using contraception  infection: there is a small increased risk of pelvic inflammatory disease in the first 20 days after insertion but after this period the risk returns to that of a standard population  expulsion: risk is around 1 in 20, and is most likely to occur in the first 3 months Q12)_A 19-year-old woman is seen the day after being discharged from hospital following a termination of pregnancy at 14 weeks. She is keen to start the combined oral contraceptive (COC) pill despite discussing long acting reversible contraceptives. What is the most appropriate action? A. Start COC immediately B. Start COC after 7 days C. Refuse to prescribe a contraceptive unless she chooses a long acting reversible contraceptive D. Start COC on first day of next period E. Start COC after 21 days The COC can be started immediately after a miscarriage or abortion. Women are protected from pregnancy straight away. Combined oral contraceptive pill: counselling Women who are considering taking the combined oral contraceptive pill (COC) should be counselled in a number of areas: Potential harms and benefits, including  the COC is > 99% effective if taken correctly  small risk of blood clots  very small risk of heart attacks and strokes  increased risk of breast cancer and cervical cancer Advice on taking the pill, including
  • 14.  if the COC is started within the first 5 days of the cycle then there is no need for additional contraception. If it is started at any other point in the cycle then alternative contraception should be used (e.g. condoms) for the first 7 days  should be taken at the same time everyday  taken for 21 days then stopped for 7 days - similar uterine bleeding to menstruation  advice that intercourse during the pill-free period is only safe if the next pack is started on time Discussion on situations here efficacy may be reduced*  if vomiting within 2 hours of taking COC pill  if taking liver enzyme inducing drugs Other information  discussion on STIs *Concurrent antibiotic use  for many years doctors in the UK have advised that the concurrent use of antibiotics may interfere with the enterohepatic circulation of oestrogen and thus make the combined oral contraceptive pill ineffective - 'extra-precautions' were advised for the duration of antibiotic treatment and for 7 days afterwards  no such precautions are taken in the US or the majority of mainland Europe  in 2011 the Faculty of Sexual & Reproductive Healthcare produced new guidelines abandoning this approach. The latest edition of the BNF has been updated in line with this guidance  precautions should still be taken with enzyme inducing antibiotics such as rifampicin Q13)_A 44-year-oldfemale has a Mirena(intrauterine system) fittedfor contraceptionon day 12 of her cycle.How longwill it take before it can be reliedupon as a methodof contraception? A. Immediately B. 2 days C. 5 days D. 7 days
  • 15. E. Until firstday of nextperiod Contraceptives - time until effective (if not first day period):  instant: IUD  2 days: POP  7 days: COC, injection, implant, IUS Intrauterine contraceptive devices Intrauterine contraceptive devices comprise both conventional copper intrauterine devices (IUDs) and levonorgestrel-releasing intrauterine systems (IUS, Mirena). The IUS is also used in the management of menorrhagia Effectiveness  both the IUD and IUS are more than 99% effective Mode of action  IUD: primary mode of action is prevention of fertilisation by causing decreased sperm motility and survival (possibly an effect of copper ions)  IUS: levonorgestrel prevents endometrial proliferation and causes cervical mucous thickening Counselling  IUD is effective immediately following insertion  IUS can be relied upon after 7 days Potential problems  IUDs make periods heavier, longer and more painful  the IUS is associated with initial frequent uterine bleeding and spotting. Later women typically have intermittent light menses with less dysmenorrhoea and some women become amenorrhoeic  uterine perforation: up to 2 per 1000 insertions
  • 16.  the proportion of pregnancies that are ectopic is increased but the absolute number of ectopic pregnancies is reduced, compared to a woman not using contraception  infection: there is a small increased risk of pelvic inflammatory disease in the first 20 days after insertion but after this period the risk returns to that of a standard population  expulsion: risk is around 1 in 20, and is most likely to occur in the first 3 months Q14)_ What is the failure rate offemale sterilisation? A. 1 in100 B. 1 in200 C. 1 in300 D. 1 in400 E. 1 in500 Female sterilisation - failure rate = 1 in 200 Sterilisation Male sterilisation - vasectomy  failure rate: 1 per 2,000*  simple operation, can be done under LA (some GA), go home after a couple of hours  doesn't work immediately  semen analysis needs to be performed twice following a vasectomy before a man can have unprotected sex (usually at 16 and 20 weeks)  complications: bruising, haematoma, infection, sperm granuloma, chronic testicular pain (affects between 5-30% men)  the success rate of vasectomy reversal is up to 55%, if done within 10 years, and approximately 25% after more than 10 years Female sterilisation  failure rate: 1 per 200*  usually done by laparoscopy under general anaesthetic  generally done as a day case
  • 17.  many different techniques involving clips (e.g. Filshie clips) , blockage, rings (Falope rings) and salpingectomy  complications: increased risk of ectopic if sterilisation fails, general complications of GA/laparoscopy  the current success rate of female sterilisation reversal is between 50-60% *source = Royal College of Obstetricians and Gynaecologists Q15)_A 44-year-old man attends for counselling with regards to a vasectomy. Which one of the following statements is true regarding vasectomy? A. Vasectomy is effective immediately B. Female sterilisation is more effective C. Two negative semen samples should be obtained at 2 and 4 weeks before other contraceptive methods are stopped D. Chronic testicular pain is seen in more than 5%of patients E. Sexual intercourse should be avoided for one month to reduce the chance of a sperm granuloma Sterilisation Male sterilisation - vasectomy  failure rate: 1 per 2,000*  simple operation, can be done under LA (some GA), go home after a couple of hours  doesn't work immediately  semen analysis needs to be performed twice following a vasectomy before a man can have unprotected sex (usually at 16 and 20 weeks)  complications: bruising, haematoma, infection, sperm granuloma, chronic testicular pain (affects between 5-30% men)  the success rate of vasectomy reversal is up to 55%, if done within 10 years, and approximately 25% after more than 10 years Female sterilisation  failure rate: 1 per 200*  usually done by laparoscopy under general anaesthetic  generally done as a day case
  • 18.  many different techniques involving clips (e.g. Filshie clips) , blockage, rings (Falope rings) and salpingectomy  complications: increased risk of ectopic if sterilisation fails, general complications of GA/laparoscopy  the current success rate of female sterilisation reversal is between 50-60% *source = Royal College of Obstetricians and Gynaecologists Q16)_ A 33-year-old woman is reviewed following a routine cervical smear. She had an intrauterine device (IUD) inserted for contraception 2 years ago. She is currently well and reports no new problems. The smear report shows no evidence of dyskaryosis but states that Actinomyces-like organisms had been identified. What is the most appropriate management? A. Remove IUD + high vaginal swab in 1 month B. No action needed C. Remove IUD + oral doxycycline D. Oral metronidazole E. Remove IUD + oral metronidazole Actinomyces-like organisms (ALOs) are a commensal of the female genital tract. Current advice from the Faculty of Sexual and Reproductive Healthcare suggests IUDs need not be removed if the patient is asymptomatic. If the patient was symptomatic then IUD removal should be considered along with penicillin therapy Intrauterine contraceptive devices Intrauterine contraceptive devices comprise both conventional copper intrauterine devices (IUDs) and levonorgestrel-releasing intrauterine systems (IUS, Mirena). The IUS is also used in the management of menorrhagia Effectiveness  both the IUD and IUS are more than 99% effective Mode of action  IUD: primary mode of action is prevention of fertilisation by causing decreased sperm motility and survival (possibly an effect of copper ions)  IUS: levonorgestrel prevents endometrial proliferation and causes cervical mucous thickening
  • 19. Counselling  IUD is effective immediately following insertion  IUS can be relied upon after 7 days Potential problems  IUDs make periods heavier, longer and more painful  the IUS is associated with initial frequent uterine bleeding and spotting. Later women typically have intermittent light menses with less dysmenorrhoea and some women become amenorrhoeic  uterine perforation: up to 2 per 1000 insertions  the proportion of pregnancies that are ectopic is increased but the absolute number of ectopic pregnancies is reduced, compared to a woman not using contraception  infection: there is a small increased risk of pelvic inflammatory disease in the first 20 days after insertion but after this period the risk returns to that of a standard population  expulsion: risk is around 1 in 20, and is most likely to occur in the first 3 months Q17)_ A female patientasks for advice about havingan intrauterine device inserted(aTT380 Slimline).Whatadvice shouldbe givenregardingthe likelyeffectonher periods? A. Periodswill tend to be longer,heavierand more painful B. Theywill stopafter6 monthsin > 50% of users C. Periodswill tendtobe lighter,shorterandlesspainful D. Continual, lightbleedingisseenin50% E. Theywill stopafter6 monthsin > 90% of users Intrauterine contraceptive devices Intrauterine contraceptive devices comprise both conventional copper intrauterine devices (IUDs) and levonorgestrel-releasing intrauterine systems (IUS, Mirena). The IUS is also used in the management of menorrhagia Effectiveness  both the IUD and IUS are more than 99% effective Mode of action
  • 20.  IUD: primary mode of action is prevention of fertilisation by causing decreased sperm motility and survival (possibly an effect of copper ions)  IUS: levonorgestrel prevents endometrial proliferation and causes cervical mucous thickening Counselling  IUD is effective immediately following insertion  IUS can be relied upon after 7 days Potential problems  IUDs make periods heavier, longer and more painful  the IUS is associated with initial frequent uterine bleeding and spotting. Later women typically have intermittent light menses with less dysmenorrhoea and some women become amenorrhoeic  uterine perforation: up to 2 per 1000 insertions  the proportion of pregnancies that are ectopic is increased but the absolute number of ectopic pregnancies is reduced, compared to a woman not using contraception  infection: there is a small increased risk of pelvic inflammatory disease in the first 20 days after insertion but after this period the risk returns to that of a standard population  expulsion: risk is around 1 in 20, and is most likely to occur in the first 3 months Q18)_ Whichone of the followingisan absolute contraindicationto combinedoral contraceptive pill use? A. Controlledhypertension B. Historyof cholestasis C. 36-year-oldwoman smoking20 cigarettes/day D. BMI of 38 kg/m^2 E. Migraine withoutaura Combined oral contraceptive pill: contraindications The decision of whether to start a women on the combined oral contraceptive pill is now guided by the UK Medical Eligibility Criteria (UKMEC). This scale categorises the potential cautions and contraindications according to a four point scale, as detailed below:
  • 21.  UKMEC 1: a condition for which there is no restriction for the use of the contraceptive method  UKMEC 2: advantages generally outweigh the disadvantages  UKMEC 3: disadvantages generally outweigh the advantages  UKMEC 4: represents an unacceptable health risk Examples of UKMEC 3 conditions include  more than 35 years old and smoking less than 15 cigarettes/day  BMI > 35 kg/m^2*  migraine without aura and more than 35 years old  family history of thromboembolic disease in first degree relatives < 45 years  controlled hypertension  immobility e.g. wheel chair use  breast feeding 6 weeks - 6 months postpartum Examples of UKMEC 4 conditions include  more than 35 years old and smoking more than 15 cigarettes/day  migraine with aura  history of thromboembolic disease or thrombogenic mutation  history of stroke or ischaemic heart disease  breast feeding < 6 weeks post-partum  uncontrolled hypertension  breast cancer  major surgery with prolonged immobilisation Diabetes mellitus diagnosed > 20 years ago is classified as UKMEC 3 or 4 depending on severity *The UKMEC 4 rating for a BMI > 40 kg/m^2 was removed in 2009. Q19)_A 22-year-old woman presents for her Depo-provera injection. She apologises as she forgot about her appointment last week. You calculate she received her last injection 12 weeks and 4 day ago. What is the most appropriate course of action? A. Do a pregnancy test today + give injection if negative B. Give injection today and no further action C. Give injection today + use condoms for 7 days + pregnancy test in 21 days
  • 22. D. Do not give injection + do pregnancy test in 21 days E. Give injection today + use condoms for 7 days Depo-provera can be given up to 14 weeks with no extra precautions See below - there is a discrepancy between the widely followed 14 week rule and the advice in the BNF. However, this answer is still consistent with the BNF guidance. Injectable contraceptives Depo Provera is the main injectable contraceptive used in the UK*. It contains medroxyprogesterone acetate 150mg. It is given via in intramuscular injection every 12 weeks. It can however be given up to 14 weeks after the last dose without the need for extra precautions** The main method of action is by inhibiting ovulation. Secondary effects include cervical mucus thickening and endometrial thinning. Disadvantages include the fact that the injection cannot be reversed once given. There is also a potential delayed return to fertility (maybe up to 12 months) Adverse effects  irregular bleeding  weight gain  may potentially increased risk of osteoporosis: should only be used in adolescents if no other method of contraception is suitable  not quickly reversible and fertility may return after a varying time *Noristerat, the other injectable contraceptive licensed in the UK, is rarely used in clinical practice. It is given every 8 weeks **the BNF gives different advice, stating a pregnancy test should be done if the interval is greater than 12 weeks and 5 days - this is however not commonly adhered to in the family planning community Q20)_A 25-year-old female presents to her GP as she has missed two consecutive Microgynon 30 pills. She has taken the Microgynon for the past 5 years and is currently 11 days into a packet of pills. Last night she had sexualintercourse with a new partner but unfortunately the condom split. What is the correct management? A. Perform a pregnancy test
  • 23. B. No action needed C. Advise condom use for next 7 days D. Emergency contraception should be offered E. Omit pill break at end of pack Updated guidance from the FSRH states the following after a woman has missed two pills: 'If you have missed two or more pills (i.e. more than 48 hours late), anywhere in the pack … continue taking the rest of the pack as usual and use an extra method of contraception for the next 7 days' For further information please consult the link to the FSRH guidelines. As this was a new partner consideration should be given to STI screening after an appropriate interval. Combined oral contraceptive pill: missed pill The advice from the Faculty of Sexual and Reproductive Healthcare (FSRH) has changed over recent years. The following recommendations are now made for women taken a combined oral contraceptive (COC) pill containing 30-35 micrograms of ethinylestradiol If 1 pill is missed (at any time in the cycle)  take a pill as soon as possible and then continue taking pills daily, one each day  no additional contraceptive protection needed If 2 or more pills missed  take a pill as soon as possible and then continue taking pills daily, one each day  the women should use condoms or abstain from sex until she has taken pills for 7 days in a row  if pills are missed in week 1 (Days 1-7): emergency contraception should be considered if she had unprotected sexin the pill-free interval or in week 1  if pills are missed in week 2 (Days 8-14): after seven consecutive days of taking the COC there is no need for emergency contraception*  if pills are missed in week 3 (Days 15-21): she should finish the pills in her current pack and start a new pack the next day; thus omitting the pill free interval *theoretically women would be protected if they took the COC in a pattern of 7 days on, 7 days off Q21)_Which one of the following is most associated with combined oral contraceptive pill use?
  • 24. A. Increased dysmenorrhoea B. Increased incidence of benign breast disease C. Worsening of acne D. Increased risk of colorectal cancer E. Increased risk of cervicalcancer Combined oral contraceptive pill  increased risk of breast and cervical cancer  protective against ovarian and endometrial cancer Combined oral contraceptive pill: advantages/disadvantages Advantages of combined oral contraceptive pill  highly effective (failure rate < 1 per 100 woman years)  doesn't interfere with sex  contraceptive effects reversible upon stopping  usually makes periods regular, lighter and less painful  reduced risk of ovarian, endometrial and colorectal cancer  may protect against pelvic inflammatory disease  may reduce ovarian cysts, benign breast disease, acne vulgaris Disadvantages of combined oral contraceptive pill  people may forget to take it  offers no protection against sexually transmitted infections  increased risk of venous thromboembolic disease  increased risk of breast and cervical cancer  increased risk of stroke and ischaemic heart disease (especially in smokers)  temporary side-effects such as headache, nausea, breast tenderness may be seen
  • 25. Whilst some users report weight gain whilst taking the combined oral contraceptive pill a Cochrane review did not support a causal relationship Q22)_A 36-year-old female starts Cerazette (desogestrel) on day 7 of her cycle. How long will it take before it can be relied upon as a method of contraception? A. Immediately B. 2 days C. 5 days D. 7 days E. Until first day of next period Contraceptives - time until effective (if not first day period):  instant: IUD  2 days: POP  7 days: COC, injection, implant, IUS Progestogen only pill: counselling Women who are considering taking the progestogen only pill (POP) should be counselled in a number of areas: Potential adverse effects  irregular vaginal bleeding is the most common problem Starting the POP  if commenced up to and including day 5 of the cycle it provides immediate protection, otherwise additional contraceptive methods (e.g. Condoms) should be used for the first 2 days  if switching from a combined oral contraceptive (COC) gives immediate protection if continued directly from the end of a pill packet (i.e. Day 21)
  • 26. Taking the POP  should be taken at same time everyday, without a pill free break (unlike the COC) Missed pills  if < 3 hours* late: continue as normal  if > 3 hours*: take missed pill as soon as possible, continue with rest of pack, extra precautions (e.g. Condoms) should be used until pill taking has been re-established for 48 hours Other potential problems  diarrhoea and vomiting: continue taking POP but assume pills have been missed - see above  antibiotics: have no effect on the POP**  liver enzyme inducers may reduce effectiveness Other information  discussion on STIs *for Cerazette (desogestrel) a 12 hour period is allowed **unless the antibiotic alters the P450 enzyme system, for example rifampicin Q23)_A 29-year-old woman who is 2 weeks postpartum consults you regarding contraception. She is interested in having an intrauterine device (IUD) inserted and asks when it could be fitted. She had a emergency caesarean section for failure to progress. What is the most appropriate advice to give? A. An IUD can be inserted 4 weeks postpartum B. An IUD can be inserted 12 months postpartum C. An IUD can be inserted today D. An IUD can be inserted 12 weeks postpartum E. An IUD is contraindicated in the long-term
  • 27. Guidelines do not suggest there is a need to wait any longer despite the caesarean section. Intrauterine contraceptive devices: insertion Very few contraindications to insertion of an intrauterine contraceptive device exist. Below are some conditions mentioned by the Faculty of Family Planning and Reproductive Health Care. Please see the link for the full list. UKMEC Category 3 (Risks outweigh benefits)*  between 48 hours and 4 weeks postpartum (increased risk of perforation)  initiation of method** in women with ovarian cancer UKMEC Category 4 (Unacceptable risk)  pregnancy  current pelvic infection, puerperal sepsis, immediate post-septic abortion  unexplained vaginal bleeding which is suspicious  uterine fibroids or uterine anatomical abnormalities distorting the uterine cavity NICE produced guidelines in 2005 on screening for sexually transmitted infections (STI) before insertion of an intrauterine contraceptive device  Chlamydia trachomatis in women at risk of STIs  Neisseria gonorrhoeae in women at risk of STIs, in areas where it is prevalent  any STIs in women who request it For women at increased risk of STIs prophylactic antibiotics should be given before inserting an intrauterine contraceptive device if testing has not yet been completed *current venous thromboembolism (on anticoagulants) has recently been downgraded from UKMEC 3 to UKMEC 1 **as opposed to continuation of the method Q24)-A 23-year-old female presents as she would like to switch from Microgynon 30 to another combined oral contraceptive (COC) due to mood swings.It is decided to start Marvelon. What advice should be given about switching her COC? A. 'Overlap' for 7 days after starting the new COC B. Stop Microgynon, wait until first day of next normal menstrual period before commencing new pill
  • 28. C. Finish the current pill packet and the start the new COC without a pill free interval and use condoms for 7 days D. Finish the current pill packet and the start the new COC without a pill free interval E. Switch at any time as the oestrogen content of the pill is the same Difficult one as the BNF and Faculty of Sexual & Reproductive Healthcare (FSRH) appear to give contradictory advice. The Clinical Effectiveness Unit of the FSRH have produced a statement that the pill free interval does not need to be omitted (please see link). The BNF however advises missing the pill free interval if the progesterone changes. Given the uncertainty it is best to follow the BNF Combined oral contraceptive pill: special situations Concurrent antibiotic use  for many years doctors in the UK have advised that the concurrent use of antibiotics may interfere with the enterohepatic circulation of oestrogen and thus make the combined oral contraceptive pill ineffective - 'extra- precautions' were advised for the duration of antibiotic treatment and for 7 days afterwards  no such precautions are taken in the US or the majority of mainland Europe  in 2011 the Faculty of Sexual & Reproductive Healthcare produced new guidelines abandoning this approach. The latest edition of the BNF has been updated in line with this guidance  precautions should still be taken with enzyme inducing antibiotics such as rifampicin Switching combined oral contraceptive pills  the BNF and Faculty of Sexual & Reproductive Healthcare (FSRH) appear to give contradictory advice. The Clinical Effectiveness Unit of the FSRH have stated in the Combined Oral Contraception guidelines that the pill free interval does not need to be omitted (please see link). The BNF however advises missing the pill free interval if the progesterone changes. Given the uncertainty it is best to follow the BNF Q25)_ A 27-year-old female presents to her GP as she missed her Cerazette pill (progestogen only) this morning and is unsure what to do. She normally takes the pill at around 0900 and it is now 1430. What advice should be given? A. Emergency contraception should be offered B. Perform a pregnancy test C. Take missed pill as soon as possible and omit pill break at end of pack
  • 29. D. Take missed pill now and no further action needed E. Take missed pill now and advise condom use until pill taking re-established for 48 hours As Cerazette has a 12-hour window this patient should take the pill now with no further action being needed Progestogen only pill: missed pill The missed pill rules for the progestogen only pill is as follows: If < 3 hours* late  continue as normal If > 3 hours*  take missed pill as soon as possible  continue with rest of pack  extra precautions (e.g. condoms) should be used until pill taking has been re-established for 48 hours *for Cerazette (desogestrel) a 12 hour period is allowed Q26)- What is the most common adverse effect experienced by women taking the progestogen only pill? A. Irregular vaginal bleeding B. Acne C. Mood swings D. Reduced libido E. Weight gain The October 2011 AKT feedback stated: 'With regard to AKT 13, knowledge about basic contraceptive issues seemed to be lacking. ' Progestogen only pill: advantages/disadvantages
  • 30. Advantages  highly effective (failure rate = 1 per 100 woman years)  doesn't interfere with sex  contraceptive effects reversible upon stopping  can be used whilst breast-feeding  can be used in situations where the combined oral contraceptive pill is contraindicated e.g. in smokers > 35 years of age and women with a history of venous thromboembolic disease Disadvantages  irregular periods: some users may not have periods whilst others may have irregular or light periods. This is the most common adverse effect  doesn't protect against sexually transmitted infections  increased incidence of functional ovarian cysts  common side-effects include breast tenderness, weight gain, acne and headaches. These symptoms generally subside after the first few months Q27)- A 45-year-old woman requests insertion of an intrauterine device (IUD). Which one of the following statements regarding the expulsion rate is correct? A. Occurs in around 1 in 200 women, and is more likely in the first 3 months B. Occurs in around 1 in 20 women, and is more likely after having the IUD for more than 3 years C. Occurs in around 1 in 500 women, and is more likely in the first 3 months D. Occurs in around 1 in 20 women, and is more likely in the first 3 months E. Occurs in around 1 in 200 women, and is more likely after having the IUD for more than 3 years Expulsion is the most common reason for IUD failure, hence the importance of checking the threads after each period Intrauterine contraceptive devices Intrauterine contraceptive devices comprise both conventional copper intrauterine devices (IUDs) and levonorgestrel-releasing intrauterine systems (IUS, Mirena). The IUS is also used in the management of menorrhagia Effectiveness
  • 31.  both the IUD and IUS are more than 99% effective Mode of action  IUD: primary mode of action is prevention of fertilisation by causing decreased sperm motility and survival (possibly an effect of copper ions)  IUS: levonorgestrel prevents endometrial proliferation and causes cervical mucous thickening Counselling  IUD is effective immediately following insertion  IUS can be relied upon after 7 days Potential problems  IUDs make periods heavier, longer and more painful  the IUS is associated with initial frequent uterine bleeding and spotting. Later women typically have intermittent light menses with less dysmenorrhoea and some women become amenorrhoeic  uterine perforation: up to 2 per 1000 insertions  the proportion of pregnancies that are ectopic is increased but the absolute number of ectopic pregnancies is reduced, compared to a woman not using contraception  infection: there is a small increased risk of pelvic inflammatory disease in the first 20 days after insertion but after this period the risk returns to that of a standard population  expulsion: risk is around 1 in 20, and is most likely to occur in the first 3 months Q28)- Which one of the following contraceptives may decrease bone mineral density in women? A. Depo Provera (injectable contraceptive) B. Progestogen only pill C. Mirena (intrauterine system) D. Implanon (implantable contraceptive) E. Combined oral contraceptive pill Injectable contraceptives
  • 32. Depo Provera is the main injectable contraceptive used in the UK*. It contains medroxyprogesterone acetate 150mg. It is given via in intramuscular injection every 12 weeks. It can however be given up to 14 weeks after the last dose without the need for extra precautions** The main method of action is by inhibiting ovulation. Secondary effects include cervical mucus thickening and endometrial thinning. Disadvantages include the fact that the injection cannot be reversed once given. There is also a potential delayed return to fertility (maybe up to 12 months) Adverse effects  irregular bleeding  weight gain  may potentially increased risk of osteoporosis: should only be used in adolescents if no other method of contraception is suitable  not quickly reversible and fertility may return after a varying time *Noristerat, the other injectable contraceptive licensed in the UK, is rarely used in clinical practice. It is given every 8 weeks **the BNF gives different advice, stating a pregnancy test should be done if the interval is greater than 12 weeks and 5 days - this is however not commonly adhered to in the family planning community Q29)_ Which one of the following is less common in women who take the combined oral contraceptive pill? A. Stroke B. Endometrial cancer C. Pulmonary embolism D. Cervical cancer E. Ischaemic heart disease Combined oral contraceptive pill  increased risk of breast and cervical cancer
  • 33.  protective against ovarian and endometrial cancer Combined oral contraceptive pill: advantages/disadvantages Advantages of combined oral contraceptive pill  highly effective (failure rate < 1 per 100 woman years)  doesn't interfere with sex  contraceptive effects reversible upon stopping  usually makes periods regular, lighter and less painful  reduced risk of ovarian, endometrial and colorectal cancer  may protect against pelvic inflammatory disease  may reduce ovarian cysts, benign breast disease, acne vulgaris Disadvantages of combined oral contraceptive pill  people may forget to take it  offers no protection against sexually transmitted infections  increased risk of venous thromboembolic disease  increased risk of breast and cervical cancer  increased risk of stroke and ischaemic heart disease (especially in smokers)  temporary side-effects such as headache, nausea, breast tenderness may be seen Whilst some users report weight gain whilst taking the combined oral contraceptive pill a Cochrane review did not support a causal relationship Q29)_ Which one of the following statements regarding the link between intrauterine devices (IUDs) and pelvic inflammatory disease (PID) is correct? A. Decreased risk in first 20 days then returns to normal B. There is no link between IUDs and PID C. Overall decreased risk throughout lifetime of IUD D. Overall increased risk throughout lifetime of IUD E. Increased risk in first 20 days then returns to normal
  • 34. The October 2011 AKT feedback stated: 'With regard to AKT 13, knowledge about basic contraceptive issues seemed to be lacking. ' Intrauterine contraceptive devices Intrauterine contraceptive devices comprise both conventional copper intrauterine devices (IUDs) and levonorgestrel-releasing intrauterine systems (IUS, Mirena). The IUS is also used in the management of menorrhagia Effectiveness  both the IUD and IUS are more than 99% effective Mode of action  IUD: primary mode of action is prevention of fertilisation by causing decreased sperm motility and survival (possibly an effect of copper ions)  IUS: levonorgestrel prevents endometrial proliferation and causes cervical mucous thickening Counselling  IUD is effective immediately following insertion  IUS can be relied upon after 7 days Potential problems  IUDs make periods heavier, longer and more painful  the IUS is associated with initial frequent uterine bleeding and spotting. Later women typically have intermittent light menses with less dysmenorrhoea and some women become amenorrhoeic  uterine perforation: up to 2 per 1000 insertions  the proportion of pregnancies that are ectopic is increased but the absolute number of ectopic pregnancies is reduced, compared to a woman not using contraception  infection: there is a small increased risk of pelvic inflammatory disease in the first 20 days after insertion but after this period the risk returns to that of a standard population  expulsion: risk is around 1 in 20, and is most likely to occur in the first 3 months
  • 35. Q30)_ Which one of the following is an absolute contraindication to the use of the progesterone only pill? A. Immobility following surgery B. Breast cancer 3 years ago C. Previous stroke D. History of antiphospholipid syndrome E. Concurrent use of rifampicin Progestogen only pill: contraindications The decision of whether to start a women a particular type of contraceptive is now guided by the UK Medical Eligibility Criteria (UKMEC). This scale categorises the potential cautions and contraindications according to a four point scale, as detailed below:  UKMEC 1: a condition for which there is no restriction for the use of the contraceptive method  UKMEC 2: advantages generally outweigh the disadvantages  UKMEC 3: disadvantages generally outweigh the advantages  UKMEC 4: represents an unacceptable health risk Examples of UKMEC 3 conditions include  active liver disease or past tumour  liver enzyme inducers  breast cancer more than 5 years ago  undiagnosed vaginal bleeding Examples of UKMEC 4 conditions include  pregnancy  breast cancer within the last 5 years Q31)_ You are considering prescribing ulipristal (EllaOne) for a woman who has presented requesting emergency contraception. How long after unprotected sexual intercourse may ulipristal be used? A. 72 hours B. 96 hours
  • 36. C. 120 hours (5 days) D. 144 hours (6 days) E. 168 hours (7 days) Ulipristal (EllaOne) - a new type of emergency hormonal contraception, can be used up to 120 hours post UPSI Emergency contraception There are two methods currently available in the UK: Emergency hormonal contraception There are now two methods of emergency hormonal contraception ('emergency pill', 'morning-after pill'); levonorgestrel and ulipristal, a progesterone receptor modulator. Levonorgestrel  should be taken as soon as possible - efficacy decreases with time  must be taken within 72 hrs of unprotected sexual intercourse (UPSI)*  single dose of levonorgestrel 1.5mg (a progesterone)  mode of action not fully understood - acts both to stop ovulation and inhibit implantation  84% effective is used within 72 hours of UPSI  levonorgestrel is safe and well tolerated. Disturbance of the current menstrual cycle is seen in a significant minority of women. Vomiting occurs in around 1%  if vomiting occurs within 2 hours then the dose should be repeated  can be used more than once in a menstrual cycle if clinically indicated Ulipristal  a progesterone receptor modulator currently marketed as EllaOne. The primary mode of action is thought to be inhibition of ovulation  30mg oral dose taken as soon as possible, no later than 120 hours after intercourse  concomitant use with levonorgestrel is not recommended  may reduce the effectiveness of combined oral contraceptive pills and progesterone only pills
  • 37.  caution should be exercised in patients with severe asthma  repeated dosing within the same menstrual cycle is not recommended Intrauterine device (IUD)  must be inserted within 5 days of UPSI, or  if a women presents after more than 5 days then an IUD may be fitted up to 5 days after the likely ovulation date  may inhibit fertilisation or implantation  prophylactic antibiotics may be given if the patient is considered to be at high-risk of sexually transmitted infection  is 99% effective regardless of where it is used in the cycle  may be left in-situ to provide long-term contraception. If the client wishes for the IUD to be removed it should be at least kept in until the next period *may be offered after this period as long as the client is aware of reduced effectiveness and unlicensed indication Q32)_ A 23-year-old female has a Nexplanon inserted on day 18 of her 28 day cycle. At what point can the Nexplanon be relied upon to provide contraception? A. After 2 days B. At the end of the next menstrual period C. Immediately D. After 7 days E. At the start of the next menstrual period Contraceptives - time until effective (if not first day period):  instant: IUD  2 days: POP  7 days: COC, injection, implant, IUS The October 2011 AKT feedback stated: 'With regard to AKT 13, knowledge about basic
  • 38. contraceptive issues seemed to be lacking. ' Implantable contraceptives Implanon is a non-biodegradable subdermal contraceptive implant which is currently being phased out and replaced by Nexplanon. From a pharmacological perspective Nexplanon is the same as Implanon. The two main differences are:  the applicator has been redesigned to try and prevent 'deep' insertions (i.e. subcutaneous/intramuscular)  it is radiopaque and therefore easier to locate if impalpable Both versions slowly releases the progestogen hormone etonogestrel. They are typically inserted in the proximal non-dominant arm, just overlying the tricep. The main mechanism of action is preventing ovulation. They also work by thickening the cervical mucus. Key points  highly effective: failure rate 0.07/100 women/year  long-acting: lasts 3 years  doesn't contain oestrogen so can be used if past history of thromboembolism, migraine etc  can be inserted immediately following a termination of pregnancy Disadvantages include  the need for a trained professional to insert and remove device  additional contraceptive methods are needed for the first 7 days if not inserted on day 1 to 5 of a woman's menstrual cycle Adverse effects  irregular/heavy bleeding is the main problem  'progestogen effects': headache, nausea, breast pain Q33)_ Which one of the following is an absolute contraindication to combined oral contraceptive pill use? A. Concurrent use of St John's Wort B. Family history of thromboembolic disease in first degree relatives < 45 years C. Immobility (e.g. wheelchair use)
  • 39. D. Migraine with aura E. Diabetes mellitus (diagnosed 11 years ago) Combined oral contraceptive pill: contraindications The decision of whether to start a women on the combined oral contraceptive pill is now guided by the UK Medical Eligibility Criteria (UKMEC). This scale categorises the potential cautions and contraindications according to a four point scale, as detailed below:  UKMEC 1: a condition for which there is no restriction for the use of the contraceptive method  UKMEC 2: advantages generally outweigh the disadvantages  UKMEC 3: disadvantages generally outweigh the advantages  UKMEC 4: represents an unacceptable health risk Examples of UKMEC 3 conditions include  more than 35 years old and smoking less than 15 cigarettes/day  BMI > 35 kg/m^2*  migraine without aura and more than 35 years old  family history of thromboembolic disease in first degree relatives < 45 years  controlled hypertension  immobility e.g. wheel chair use  breast feeding 6 weeks - 6 months postpartum Examples of UKMEC 4 conditions include  more than 35 years old and smoking more than 15 cigarettes/day  migraine with aura  history of thromboembolic disease or thrombogenic mutation  history of stroke or ischaemic heart disease  breast feeding < 6 weeks post-partum  uncontrolled hypertension  breast cancer  major surgery with prolonged immobilisation Diabetes mellitus diagnosed > 20 years ago is classified as UKMEC 3 or 4 depending on severity *The UKMEC 4 rating for a BMI > 40 kg/m^2 was removed in 2009.
  • 40. Q34)_ Which one of the following is an absolute contraindication to combined oral contraceptive pill use? A. 37-year-old woman smoking 10 cigarettes/day B. 4 weeks post-partum and breast feeding C. Being a wheelchair user D. BMI of 43 kg / m^2 E. Family history of thromboembolic disease in first degree relatives < 45 years Option A, C, D & E are relative contraindications (UKMEC 3) whilst option B is an absolute contraindication (UKMEC 4). The UKMEC 4 rating for a BMI > 40 kg/m^2 was removed in 2009. Combined oral contraceptive pill: contraindications The decision of whether to start a women on the combined oral contraceptive pill is now guided by the UK Medical Eligibility Criteria (UKMEC). This scale categorises the potential cautions and contraindications according to a four point scale, as detailed below:  UKMEC 1: a condition for which there is no restriction for the use of the contraceptive method  UKMEC 2: advantages generally outweigh the disadvantages  UKMEC 3: disadvantages generally outweigh the advantages  UKMEC 4: represents an unacceptable health risk Examples of UKMEC 3 conditions include  more than 35 years old and smoking less than 15 cigarettes/day  BMI > 35 kg/m^2*  migraine without aura and more than 35 years old  family history of thromboembolic disease in first degree relatives < 45 years  controlled hypertension  immobility e.g. wheel chair use  breast feeding 6 weeks - 6 months postpartum Examples of UKMEC 4 conditions include  more than 35 years old and smoking more than 15 cigarettes/day  migraine with aura  history of thromboembolic disease or thrombogenic mutation
  • 41.  history of stroke or ischaemic heart disease  breast feeding < 6 weeks post-partum  uncontrolled hypertension  breast cancer  major surgery with prolonged immobilisation Diabetes mellitus diagnosed > 20 years ago is classified as UKMEC 3 or 4 depending on severity *The UKMEC 4 rating for a BMI > 40 kg/m^2 was removed in 2009. Q35)_ What is the main mechanism of action of Cerazette (desogestrel)? A. Thickens cervical mucous B. Inhibits ovulation C. Causes endometrial thinning D. Inhibits implantation E. Toxic to sperm Cerazette inhibits ovulation in around 97-99% of cycles. Progestogen only pill: types Second generation  norethisterone  levonorgestrel  ethynodiol diacetate Third generation  desogestrel (Cerazette) Cerazette  new third generation type of progestogen only pill (POP) containing desogestrel  inhibits ovulation in the majority of women  users can take the pill up to 12 hours late rather than 3 hours like other POPs
  • 42. Q36)_ Each one of the following would decrease the effectiveness of the combined oral contraceptive pill, except: A. Rifampicin B. Sodium valproate C. Carbamazepine D. St John's Wort E. Phenytoin P450 enzyme system Induction usually requires prolonged exposure to the inducing drug, as opposed to P450 inhibitors, where effects are often seen rapidly Inducers of the P450 system include  antiepileptics: phenytoin, carbamazepine  barbiturates: phenobarbitone  rifampicin  St John's Wort  chronic alcohol intake  griseofulvin  smoking (affects CYP1A2, reason why smokers require more aminophylline) Inhibitors of the P450 system include  antibiotics: ciprofloxacin, erythromycin  isoniazid  cimetidine, omeprazole  amiodarone  allopurinol  imidazoles: ketoconazole, fluconazole  SSRIs: fluoxetine, sertraline  ritonavir  sodium valproate  acute alcohol intake  quinupristin Q37)- What is the main mechanism of action of Nexplanon (etonogestrelcontraceptive implant)?
  • 43. A. Thickens cervical mucus B. Thins endometrial lining C. Inhibition of ovulation D. Causes fallopian tube dysfunction E. Prevents implantation Nexplanon - main mechanism of action is inhibition of ovulation Implantable contraceptives Implanon is a non-biodegradable subdermal contraceptive implant which is currently being phased out and replaced by Nexplanon. From a pharmacological perspective Nexplanon is the same as Implanon. The two main differences are:  the applicator has been redesigned to try and prevent 'deep' insertions (i.e. subcutaneous/intramuscular)  it is radiopaque and therefore easier to locate if impalpable Both versions slowly releases the progestogen hormone etonogestrel. They are typically inserted in the proximal non-dominant arm, just overlying the tricep. The main mechanism of action is preventing ovulation. They also work by thickening the cervical mucus. Key points  highly effective: failure rate 0.07/100 women/year  long-acting: lasts 3 years  doesn't contain oestrogen so can be used if past history of thromboembolism, migraine etc  can be inserted immediately following a termination of pregnancy Disadvantages include  the need for a trained professional to insert and remove device  additional contraceptive methods are needed for the first 7 days if not inserted on day 1 to 5 of a woman's menstrual cycle Adverse effects
  • 44.  irregular/heavy bleeding is the main problem  'progestogen effects': headache, nausea, breast pain Q38)_ A 16-year-old female with a history of acne requests to start a combined oral contraceptive pill (COC). She has been taking oxytetracycline for the past 2 months. What is the most appropriate advice? A. A double-dose COC should be used B. Condoms should be used for the first 14 days of COC use C. She can start using a COC with usual advice D. A COC is inappropriate and an alternative method of contraception should be considered E. Condoms should be used for the first 21 days of COC use The usual rules regarding starting the COC should still be adhered to, i.e. use condoms for 7 days if not started on the first day of next period Combined oral contraceptive pill: special situations Concurrent antibiotic use  for many years doctors in the UK have advised that the concurrent use of antibiotics may interfere with the enterohepatic circulation of oestrogen and thus make the combined oral contraceptive pill ineffective - 'extra- precautions' were advised for the duration of antibiotic treatment and for 7 days afterwards  no such precautions are taken in the US or the majority of mainland Europe  in 2011 the Faculty of Sexual & Reproductive Healthcare produced new guidelines abandoning this approach. The latest edition of the BNF has been updated in line with this guidance  precautions should still be taken with enzyme inducing antibiotics such as rifampicin Switching combined oral contraceptive pills  the BNF and Faculty of Sexual & Reproductive Healthcare (FSRH) appear to give contradictory advice. The Clinical Effectiveness Unit of the FSRH have stated in the Combined Oral Contraception guidelines that the pill free interval does not need to be omitted (please see link). The BNF however advises missing the pill free interval if the progesterone changes. Given the uncertainty it is best to follow the BNF
  • 45. Q39)_ A 33-year-old obese woman presents to surgery requesting advice about contraception. Her body mass index is 36 kg/m^2. What is the most suitable prescription? A. Desogestrel (Cerazette) 150 mcg od (double dose) B. Microgynon 30, two tablets a day as directed C. Microgynon 30, one tablet a day as directed D. Norethisterone (Noriday) 5mg tds E. Desogestrel (Cerazette) 75 mcg od (standard dose) There is little evidence to support the practice of prescribing double the dose of POP in obese woman. Whilst some practitioners may err on the side of caution and prescribe double the dose of POP for woman using older preparations this is not justifiable with Cerazette, given the high plasma levels achieved with this drug. Progestogen only pill: counselling Women who are considering taking the progestogen only pill (POP) should be counselled in a number of areas: Potential adverse effects  irregular vaginal bleeding is the most common problem Starting the POP  if commenced up to and including day 5 of the cycle it provides immediate protection, otherwise additional contraceptive methods (e.g. Condoms) should be used for the first 2 days  if switching from a combined oral contraceptive (COC) gives immediate protection if continued directly from the end of a pill packet (i.e. Day 21) Taking the POP  should be taken at same time everyday, without a pill free break (unlike the COC) Missed pills  if < 3 hours* late: continue as normal
  • 46.  if > 3 hours*: take missed pill as soon as possible, continue with rest of pack, extra precautions (e.g. Condoms) should be used until pill taking has been re-established for 48 hours Other potential problems  diarrhoea and vomiting: continue taking POP but assume pills have been missed - see above  antibiotics: have no effect on the POP**  liver enzyme inducers may reduce effectiveness Other information  discussion on STIs *for Cerazette (desogestrel) a 12 hour period is allowed **unless the antibiotic alters the P450 enzyme system, for example rifampicin. Q40)_ A woman rings for advice 4 days post-partum. She is keen to start her progestogen- only pill again. There have been no problems since giving birth and breast feeding is going well. What is the most appropriate advice? A. Contraindicated if breast-feeding B. Start on day 21, effective immediately C. Start immediately, effective immediately D. Start on day 21, effective after 2 days E. Start on day 7, effective immediately Post-partum contraception Progestogen only pill (POP)  may be started from day 21 - this will provide immediate contraception. If used earlier may cause breakthrough bleeding  after day 21 additional contraception should be used for the first 2 days  a small amount of progestogen enters breast milk but this is not harmful to the infant
  • 47. Combined oral contraceptive pill (COC)  absolutely contraindicated - UKMEC 4 - if breast feeding < 6 weeks post-partum  relatively contraindicated - UKMEC 3 - if breast feeding 6 weeks - 6 months postpartum  the COC may reduce breast milk production in lactating mothers  may be started from day 21 - this will provide immediate contraception  after day 21 additional contraception should be used for the first 7 days Q41)_ Concurrent use of which one of the following would make combined oral contraceptive pill less effective? A. Fluconazole B. Sodium valproate C. Allopurinol D. Isoniazid E. Carbamazepine P450 enzyme system Induction usually requires prolonged exposure to the inducing drug, as opposed to P450 inhibitors, where effects are often seen rapidly Inducers of the P450 system include  antiepileptics: phenytoin, carbamazepine  barbiturates: phenobarbitone  rifampicin  St John's Wort  chronic alcohol intake  griseofulvin  smoking (affects CYP1A2, reason why smokers require more aminophylline) Inhibitors of the P450 system include  antibiotics: ciprofloxacin, erythromycin  isoniazid  cimetidine, omeprazole  amiodarone  allopurinol
  • 48.  imidazoles: ketoconazole, fluconazole  SSRIs: fluoxetine, sertraline  ritonavir  sodium valproate  acute alcohol intake  quinupristin Q42)_ Concurrent use of which one of the following would make the combined oral contraceptive pill less effective? A. Fluconazole B. Cimetidine C. St John's Wort D. Fluoxetine E. Isoniazid St John's Wort is an inducers of the P450 enzyme system in the liver. This results in the combined oral contraceptive pill being metabolised faster and hence may reduce effectiveness. P450 enzyme system Induction usually requires prolonged exposure to the inducing drug, as opposed to P450 inhibitors, where effects are often seen rapidly Inducers of the P450 system include  antiepileptics: phenytoin, carbamazepine  barbiturates: phenobarbitone  rifampicin  St John's Wort  chronic alcohol intake  griseofulvin  smoking (affects CYP1A2, reason why smokers require more aminophylline) Inhibitors of the P450 system include  antibiotics: ciprofloxacin, erythromycin  isoniazid  cimetidine, omeprazole  amiodarone
  • 49.  allopurinol  imidazoles: ketoconazole, fluconazole  SSRIs: fluoxetine, sertraline  ritonavir  sodium valproate  acute alcohol intake  quinupristin Q43)_ A 17-year-old female presents requesting advice as she forgot to take her Microgynon 30 pills on a weekend away. She is normally very good at remembering her pill but has missed days 10, 11 and 12 of her packet and it is now day 13. Although she took the day 13 pill this morning she is concerned she may become pregnant and she had unprotected sexual intercourse whilst away. What is the most appropriate management? A. No action needed B. No action needed but omit pill break at end of pack C. Offer emergency contraception - hormonal D. Offer emergency contraception - intrauterine device E. No action needed but use condoms for next 7 days Tough question. As the patient had taken the pill for 7 days in a row previously she is protected for the next 7 days. The FSRH guidelines state: 'after seven consecutive pills have been taken there is no need for emergency contraception' - please consult the link. The guidelines also recommend in this situation using condoms for the next 7 days. The October 2011 AKT feedback stated: 'With regard to AKT 13, knowledge about basic contraceptive issues seemed to be lacking. ' Combined oral contraceptive pill: missed pill The advice from the Faculty of Sexual and Reproductive Healthcare (FSRH) has changed over recent years. The following recommendations are now made for women taken a combined oral contraceptive (COC) pill containing 30-35 micrograms of ethinylestradiol If 1 pill is missed (at any time in the cycle)  take a pill as soon as possible and then continue taking pills daily, one each day  no additional contraceptive protection needed If 2 or more pills missed  take a pill as soon as possible and then continue taking pills daily, one each day
  • 50.  the women should use condoms or abstain from sex until she has taken pills for 7 days in a row  if pills are missed in week 1 (Days 1-7): emergency contraception should be considered if she had unprotected sexin the pill-free interval or in week 1  if pills are missed in week 2 (Days 8-14): after seven consecutive days of taking the COC there is no need for emergency contraception*  if pills are missed in week 3 (Days 15-21): she should finish the pills in her current pack and start a new pack the next day; thus omitting the pill free interval *theoretically women would be protected if they took the COC in a pattern of 7 days on, 7 days off Q44)_ Which one of the following is an absolute contraindication to combined oral contraceptive pill use? A. Known thrombogenic mutation B. Family history of thromboembolic disease in first degree relatives < 45 years C. Hyperlipidaemia D. Concurrent use of rifampicin E. Immobility (e.g. wheelchair use). Combined oral contraceptive pill: contraindications The decision of whether to start a women on the combined oral contraceptive pill is now guided by the UK Medical Eligibility Criteria (UKMEC). This scale categorises the potential cautions and contraindications according to a four point scale, as detailed below:  UKMEC 1: a condition for which there is no restriction for the use of the contraceptive method  UKMEC 2: advantages generally outweigh the disadvantages  UKMEC 3: disadvantages generally outweigh the advantages  UKMEC 4: represents an unacceptable health risk Examples of UKMEC 3 conditions include  more than 35 years old and smoking less than 15 cigarettes/day  BMI > 35 kg/m^2*  migraine without aura and more than 35 years old
  • 51.  family history of thromboembolic disease in first degree relatives < 45 years  controlled hypertension  immobility e.g. wheel chair use  breast feeding 6 weeks - 6 months postpartum Examples of UKMEC 4 conditions include  more than 35 years old and smoking more than 15 cigarettes/day  migraine with aura  history of thromboembolic disease or thrombogenic mutation  history of stroke or ischaemic heart disease  breast feeding < 6 weeks post-partum  uncontrolled hypertension  breast cancer  major surgery with prolonged immobilisation Diabetes mellitus diagnosed > 20 years ago is classified as UKMEC 3 or 4 depending on severity *The UKMEC 4 rating for a BMI > 40 kg/m^2 was removed in 2009. Q45)_ Which one of the following statements regarding the use of ulipristal (EllaOne) for emergency contraception is true? A. Should be used concomitantly with levonorgestrel to increase efficacy B. Can be used more than once in a menstrual cycle if clinically indicated C. May reduce the effectiveness of combined oral contraceptive pills D. Is more effective than an intrauterine device if used before 72 hours has elapsed E. Two doses are taken 12 hours apart Emergency contraception There are two methods currently available in the UK: Emergency hormonal contraception There are now two methods of emergency hormonal contraception ('emergency pill', 'morning-after pill'); levonorgestrel and ulipristal, a progesterone receptor modulator.
  • 52. Levonorgestrel  should be taken as soon as possible - efficacy decreases with time  must be taken within 72 hrs of unprotected sexual intercourse (UPSI)*  single dose of levonorgestrel 1.5mg (a progesterone)  mode of action not fully understood - acts both to stop ovulation and inhibit implantation  84% effective is used within 72 hours of UPSI  levonorgestrel is safe and well tolerated. Disturbance of the current menstrual cycle is seen in a significant minority of women. Vomiting occurs in around 1%  if vomiting occurs within 2 hours then the dose should be repeated  can be used more than once in a menstrual cycle if clinically indicated Ulipristal  a progesterone receptor modulator currently marketed as EllaOne. The primary mode of action is thought to be inhibition of ovulation  30mg oral dose taken as soon as possible, no later than 120 hours after intercourse  concomitant use with levonorgestrel is not recommended  may reduce the effectiveness of combined oral contraceptive pills and progesterone only pills  caution should be exercised in patients with severe asthma  repeated dosing within the same menstrual cycle is not recommended Intrauterine device (IUD)  must be inserted within 5 days of UPSI, or  if a women presents after more than 5 days then an IUD may be fitted up to 5 days after the likely ovulation date  may inhibit fertilisation or implantation  prophylactic antibiotics may be given if the patient is considered to be at high-risk of sexually transmitted infection  is 99% effective regardless of where it is used in the cycle  may be left in-situ to provide long-term contraception. If the client wishes for the IUD to be removed it should be at least kept in until the next period *may be offered after this period as long as the client is aware of reduced effectiveness and unlicensed indication
  • 53. Q46)_A 19-year-old woman elects to have an implantable contraceptive. What is the main advantage of Nexplanon over Implanon? A. Length of contraceptive effect increased from 3 to 5 years B. Works immediately regardless of when inserted C. Less irregular bleeding D. No local anaesthetic required for insertion E. New design makes the insertion of implants that are too deep less likely Implantable contraceptives Implanon is a non-biodegradable subdermal contraceptive implant which is currently being phased out and replaced by Nexplanon. From a pharmacological perspective Nexplanon is the same as Implanon. The two main differences are:  the applicator has been redesigned to try and prevent 'deep' insertions (i.e. subcutaneous/intramuscular)  it is radiopaque and therefore easier to locate if impalpable Both versions slowly releases the progestogen hormone etonogestrel. They are typically inserted in the proximal non-dominant arm, just overlying the tricep. The main mechanism of action is preventing ovulation. They also work by thickening the cervical mucus. Key points  highly effective: failure rate 0.07/100 women/year  long-acting: lasts 3 years  doesn't contain oestrogen so can be used if past history of thromboembolism, migraine etc  can be inserted immediately following a termination of pregnancy Disadvantages include  the need for a trained professional to insert and remove device  additional contraceptive methods are needed for the first 7 days if not inserted on day 1 to 5 of a woman's menstrual cycle Adverse effects  irregular/heavy bleeding is the main problem
  • 54.  'progestogen effects': headache, nausea, breast pain Q47)_ You are considering offering one of your female patients the intrauterine system (Mirena) as a method of contraception. Which one of the following is most likely to represent a contraindication to this? A. Epilepsy B. Being 5 weeks post-partum C. Current treatment for ovarian cancer D. Past history of ectopic pregnancy E. Past history of pelvic inflammatory disease Intrauterine contraceptive devices: insertion Very few contraindications to insertion of an intrauterine contraceptive device exist. Below are some conditions mentioned by the Faculty of Family Planning and Reproductive Health Care. Please see the link for the full list. UKMEC Category 3 (Risks outweigh benefits)*  between 48 hours and 4 weeks postpartum (increased risk of perforation)  initiation of method** in women with ovarian cancer UKMEC Category 4 (Unacceptable risk)  pregnancy  current pelvic infection, puerperal sepsis, immediate post-septic abortion  unexplained vaginal bleeding which is suspicious  uterine fibroids or uterine anatomical abnormalities distorting the uterine cavity NICE produced guidelines in 2005 on screening for sexually transmitted infections (STI) before insertion of an intrauterine contraceptive device  Chlamydia trachomatis in women at risk of STIs  Neisseria gonorrhoeae in women at risk of STIs, in areas where it is prevalent  any STIs in women who request it For women at increased risk of STIs prophylactic antibiotics should be given before inserting an intrauterine contraceptive device if testing has not yet been completed
  • 55. *current venous thromboembolism (on anticoagulants) has recently been downgraded from UKMEC 3 to UKMEC 1 **as opposed to continuation of the method
  • 56. Womens Health ( MRCGP Qs ) : Gynecology. Q1-A 17-year-old girl presents due to painful periods. These have been present for the past three years and are associated with a normal amount of blood loss. Her periods are regular and there is no abnormal bleeding. She is not yet sexually active. What is the most appropriate first-line treatment? A. Tranexamic acid B. Referral for relaxation therapy C. Paracetamol D. Combined oral contraceptive pill E. Ibuprofen NSAIDs are offered first-line as they will inhibit prostaglandin synthesis, one of the main causes of dysmenorrhoea pains. Dysmenorrhoea Dysmenorrhoea is characterised by excessive pain during the menstrual period. It is traditionally divided into primary and secondary dysmenorrhoea. Primary dysmenorrhoea In primary dysmenorrhoea there is no underlying pelvic pathology. It affects up to 50% of menstruating women and usually appears within 1-2 years of the menarche. Excessive endometrial prostaglandin production is thought to be partially responsible. Features  pain typically starts just before or within a few hours of the period starting  suprapubic cramping pains which may radiate to the back or down the thigh Management  NSAIDs such as mefenamic acid and ibuprofen are effective in up to 80% of women. They work by inhibiting prostaglandin production  combined oral contraceptive pills are used second line Secondary dysmenorrhoea Secondary dysmenorrhoea typically develops many years after the menarche and is the result of an
  • 57. underlying pathology. In contrast to primary dysmenorrhoea the pain usually starts 3-4 days before the onset of the period. Causes include:  endometriosis  adenomyosis  pelvic inflammatory disease  intrauterine devices*  fibroids Clinical Knowledge Summaries recommend referring all patients with secondary dysmenorrhoea to gynaecology for investigation. *this refers to normal copper coils. Note that the intrauterine system (Mirena) may help dysmenorrhoea Q2)_ A 57-year-old female presents due to problems with urine leakage over the past six months. She describes frequent voiding and not always being able to get to the toilet in time. She denies losing urine when coughing or sneezing.What is the most appropriate initial treatment? A. Trial of oxybutynin B. Bladder retraining C. Regular toileting D. Pelvic floor muscle training E. Topical oestrogen cream Urinary incontinence - first-line treatment:  urge incontinence: bladder retraining  stress incontinence: pelvic floor muscle training Urinary incontinence Urinary incontinence (UI) is a common problem, affecting around 4-5% of the UK population. It is more common in elderly females. NICE released guidance on the management of UI in 2006 Causes
  • 58.  overactive bladder (OAB)/urge incontinence: due to detrusor over activity  stress incontinence: leaking small amounts when coughing or laughing  mixed incontinence: both urge and stress  overflow incontinence: due to bladder outlet obstruction, e.g. due to prostate enlargement Initial investigation  bladder diaries should be completed for a minimum of 3 days  urine dipstick and culture Management depends on whether urge or stress UI is the predominant picture. If urge incontinence is predominant:  bladder retraining (lasts for a minimum of 6 weeks, the idea is to gradually increase the intervals between voiding)  bladder stabilising drugs: immediate release oxybutynin is first-line  surgical management: e.g. sacral nerve stimulation If stress incontinence is predominant:  pelvic floor muscle training (for a minimum of 3 months)  surgical procedures: e.g. retropubic mid-urethral tape procedures Q3)_ A 49-year-old female consults her GP asking about hormone replacement therapy (HRT). What is the most compelling indication for starting HRT? A. Prevention of ischaemic heart disease B. Prevention of osteoporosis C. Reversal of vaginal atrophy D. Control of vasomotor symptoms such as flushing E. Prevention of Alzheimer's disease Main indication for HRT: control of vasomotor symptoms
  • 59. Hormone replacement therapy: indications Hormone replacement therapy (HRT) involves the use of a small dose of oestrogen, combined with a progestogen (in women with a uterus), to help alleviate menopausal symptoms. The indications for HRT have changed significantly over the past ten years as the long-term risks became apparent, primarily as a result of the Women's Health Initiative (WHI) study. Indications  vasomotor symptoms such as flushing, insomnia and headaches  premature menopause: should be continued until the age of 50 years  osteoporosis: but should only be used as second-line treatment The main indication is the control of vasomotor symptoms. The other indications such as reversal of vaginal atrophy and prevention of osteoporosis should be treated with other agents as first-line therapies Other benefits include a reduced incidence of colorectal cancer Q4)_ A 24-year-old woman presents for review complaining of heavy periods. This has been a problem for a number of years now.She has a 28 day cycle and has heavy bleeding for 5 days. There is no intermenstrual or post-coital bleeding. General and gynaecological examination is unremarkable. What is the minimum set of investigations that this patient should be offered? A. FBC B. FBC + ferritin C. No investigations if the gynaecological examination is normal D. FBC + pelvic ultrasound E. FBC + TFT Menorrhagia - all women should have a FBC Menorrhagia: management Menorrhagia was previously defined as total blood loss > 80 ml per menses, but it is obviously
  • 60. difficult to quantify. The management has therefore shifted towards what the woman considers to be excessive. Prior to the 1990's many women underwent a hysterectomy to treat heavy periods but since that time the approach has altered radically. The management of menorrhagia now depends on whether a women needs contraception. Investigations  a full blood count should be performed in all women  further investigations are based upon the history and examination findings Does not require contraception  either mefenamic acid 500 mg tds (particularly if there is dysmenorrhoea as well) or tranexamic acid 1 g tds. Both are started on the first day of the period  if no improvement then try other drug whilst awaiting referral Requires contraception, options include  intrauterine system (Mirena) should be considered first-line  combined oral contraceptive pill  long-acting progestogens Norethisterone 5 mg tds can be used as a short-term option to rapidly stop heavy menstrual bleeding. Q5)_ A 31-year-old woman presents as she has noted an offensive, fishy vaginal discharge. She describes a grey, watery discharge. What is the most likely diagnosis? A. Trichomonas vaginalis B. Candida C. Chlamydia D. Bacterial vaginosis E. Physiological discharge Vaginal discharge Vaginal discharge is a common presenting symptom and is not always pathological Common causes
  • 61.  physiological  Candida  Trichomonas vaginalis  bacterial vaginosis Less common causes  whilst cervical infections such as Chlamydia and Gonorrhoea can cause a vaginal discharge this is rarely the presenting symptoms  ectropion  foreign body  cervical cancer Key features of the common causes are listed below Condition Key features Candida 'Cottage cheese'discharge Vulvitis Itch Trichomonas vaginalis Offensive, yellow/green, frothy discharge Vulvovaginitis Strawberry cervix Bacterial vaginosis Offensive, thin, white/grey, 'fishy' discharge Q6)- A 53-year-old woman presents with urgency and frequency. Two weeks ago she consulted with a colleague as she felt 'dry' during intercourse. She has been treated for urinary tract infections on multiple occasions in the past but urine culture is always negative. Her only medication is cyclical hormone replacement therapy. A vaginal examination is performed which shows no evidence of vaginal atrophy and no masses are felt. An ultrasound is requested: Both kidneys, spleen and liver are normal size. Outline of the bladder normal. 3 cm simple ovarian cyst noted on left ovary. Right ovary and uterus normal What is the most appropriate next step? A. Refer for urodynamics B. Pelvic floor muscle training C. Trial topical oestrogen
  • 62. D. Urgent referral to gynaecology E. Refer for bladder retraining Any ovarian mass in a post-menopausal woman needs to be investigated. Ovarian enlargement: management The initial imaging modality for suspected ovarian cysts/tumours is ultrasound. The report will usually report that the cyst is either:  simple: unilocular, more likely to be physiological or benign  complex: multilocular, more likely to be malignant Management depends on the age of the patient and whether the patient is symptomatic. It should be remembered that the diagnosis of ovarian cancer is often delayed due to a vague presentation. Premenopausal women  a conservative approach may be taken for younger women (especially if < 35 years) as malignancy is less common. If the cyst is small (e.g. < 5 cm) and reported as 'simple' then it is highly likely to be benign. A repeat ultrasound should be arranged for 8-12 weeks and referral considered if it persists. Postmenopausal women  by definition physiological cysts are unlikely  any postmenopausal woman with an ovarian cyst regardless of nature or size should be referred to gynaecology for assessment Q7)_A 34-year-old woman is reviewed in surgery. She complains of a long history of deep pain during intercourse and painful periods. There is a previous history of lower abdominal pain and in the past she has been diagnosed with irritable bowel syndrome. She is upset because she would like to start a family but the pain is putting her off sex. What is the most suitable management? A. Trial of combined oral contraceptive pill B. Discuss benefits of intrauterine system C. Refer to gynaecology D. Refer for psychosexual counselling E. Arrange pelvic ultrasound
  • 63. The combination of deep dyspareunia and lower abdominal pain make a diagnosis of endometriosis likely. Initial treatment options such as the combined pill are not an option in a woman trying to conceive. For a definitive diagnosis the patient should ideally have a laparoscopy. A pelvic ultrasound is not the investigation of choice in endometriosis and may be normal with mild-moderate disease Endometriosis Endometriosis is a common condition characterised by the growth of ectopic endometrial tissue outside of the uterine cavity. Up to 10-15% of women have a degree of endometriosis Clinical features  chronic pelvic pain  dysmenorrhoea - pain often starts days before bleeding  deep dyspareunia  subfertility Less common features  urinary symptoms e.g. dysuria, urgency  dyschezia (painful bowel movements) Investigation  laparoscopy is the gold-standard investigation  there is little role for investigation in primary care (e.g. ultrasound)- if the symptoms are significant the patient should be referred for a definitive diagnosis Management depends on clinical features - there is poor correlation between laparoscopic findings and severity of symptoms  NSAIDs and other analgesia for symptomatic relief  combined oral contraceptive pill  progestogens e.g. medroxyprogesterone acetate  gonadotrophin-releasing hormone (GnRH) analogues - said to induce a 'pseudomenopause' due to the low oestrogen levels  intrauterine system (Mirena)  drug therapy unfortunately does not seem to have a significant impact on fertility rates Surgery
  • 64.  some treatments such as laparoscopic excision and laser treatment of endometriotic ovarian cysts may improve fertility Q8-9-10)_ Theme: Pelvic pain A. Appendicitis B. Ovarian torsion C. Urogenital prolapse D. Endometriosis E. Urinary tract infection F. Ovarian cyst G. Threatened miscarriage H. Irritable bowel syndrome I. Pelvic inflammatory disease J. Ectopic pregnancy For each one of the following scenarios please select the most likely diagnosis: 8. A 24-year-old woman presents with crampy suprapubic pain and light vaginal bleeding. Her last period was 9 weeks ago. Vaginal examination shows a small amount of blood around the cervix but is otherwise unremarkable. Threatened miscarriage Ectopic pregnancy should also be considered here as a number of women will not have the typical findings of cervical excitation. 9. A 67-year-old woman presents with a heavy, dragging sensation in the suprapubic region. She also has frequency and urgency. Urogenital prolapse Women who have a urogenital prolapse typically describe a 'bearing down', 'heaviness' or 'dragging' sensation. 10. A 29-year-old woman presents with suprapubic pain, irregular periods, dysuria and pain during intercourse. There is cervical excitation on examination. Pelvic inflammatory disease Cervical excitation is found in both pelvic inflammatory disease and ectopic pregnancy. Pelvic pain In women the most common cause of pelvic pain is primary dysmenorrhoea. Some women also experience transient pain in the middle of their cycle secondary to ovulation (mittelschmerz). The table below gives characteristic features for other conditions causing pelvic pain: Usually acute Ectopic pregnancy A typical historyis a female with a history of 6-8 weeks amenorrhoea who presents with
  • 65. lower abdominal pain and later develops vaginal bleeding Shoulder tip pain and cervical excitation may be seen Urinary tract infection Dysuria and frequency are common butwomen mayexperience suprapubic burning secondaryto cystitis Appendicitis Pain initial in the central abdomen before localising to the right iliac fossa Anorexia is common Tachycardia, low-grade pyrexia, tenderness in RIF Rovsing's sign:more pain in RIF than LIF when palpating LIF Pelvic inflammatory disease Pelvic pain, fever, deep dyspareunia,vaginal discharge,dysuria and menstrual irregularities may occur Cervical excitation may be found on examination Ovarian torsion Usuallysudden onsetunilateral lower abdominal pain.Onsetmaycoincide with exercise. Nausea and vomiting are common Unilateral,tender adnexal mass on examination Miscarriage Vaginal bleeding and crampylower abdominal pain following a period ofamenorrhoea Usually chronic Endometriosis Chronic pelvic pain Dysmenorrhoea - pain often starts days before bleeding Deep dyspareunia Subfertility Irritable bowel syndrome Extremely common.The most consistentfeatures are abdominal pain,bloating and change in bowel habit Features such as lethargy, nausea,backache and bladder symptoms mayalso be present Ovarian cyst Unilateral dull ache which maybe intermittentor only occur during intercourse.Torsion or rupture may lead to severe abdominal pain Large cysts may cause abdominal swelling or pressure effects on the bladder Urogenital prolapse Seen in older women Sensation ofpressure,heaviness,'bearing-down' Urinary symptoms:incontinence,frequency,urgency Q11)_ A 47-year-old woman presents to surgery seeking advice regarding contraception. She has recently started in a new relationship but is unsure if she requires contraception, as she thinks she may be going through the menopause. She is e xperiencing hot flushes and her last period was 7 months ago. What is the most appropriate advice? A. Contraception is needed until 36 months after her last period B. Contraception is needed until 18 months after her last period C. Contraception is needed until 24 months after her last period
  • 66. D. Contraception is needed until 12 months after her last period E. She no longer requires contraception Need for contraception after the menopause  12 months after the last period in women > 50 years  24 months after the last period in women < 50 years Menopause The average women in the UK goes through the menopause when she is 51 years old. The climacteric is the period prior to the menopause where women may experience symptoms, as ovarian function starts to fail Diagnosis  12 months after the last period in women > 50 years  24 months after the last period in women < 50 years It is recommend to use effective contraception until the diagnosis has been confirmed using the above criteria Q12)_You receive the results of a 29-year-old female who has recently had a routine cervical smear. Her last smear 4 years ago was reported as normal. The results are reported as follows: Moderate dyskaryosis What is the most appropriate management? A. Repeat smear in 6 months B. Repeat smear immediately C. Refer to a gynaecological oncologist D. Repeat smear in 3 months E. Refer to colposcopy
  • 67. Cervical cancer screening: interpretation of results The table below outlines the management of abnormal cervical smears (around 5% of all smears). Cervical intraepithelial neoplasia is abbreviated to CIN Mild dyskaryosis Consistentwith CIN I. Previouslywomen were offered a repeatsmear after 6 months and referral for colposcopyifchanges persisted Whilstthis is still acceptable itis now considered bestpractice to refer women immediatelyfor colposcopy Moderate dyskaryosis Consistentwith CIN II. Refer for colposcopy Severe dyskaryosis Consistentwith CIN III. Refer for colposcopy Inadequate Repeatsmear - if persistent(3 inadequate samples),assessmentbycolposcopy Q13-14-15)-Theme: Ovarian cysts A. Dermoid cyst (teratoma) B. Endometriotic cyst C. Granulosa cell tumour D. Clear cell tumour E. Corpus luteum cyst F. Mucinous cystadenoma G. Follicular cyst H. Serous cystadenoma I. Dysgerminoma J. Fibroma For each one of the following please select the answer from the list above: 13. Most common type of ovarian pathology associated with Meigs' syndrome The correct answer is Fibroma Meigs' syndrome is a benign ovarian tumour (usually a fibroma) associated with ascites and pleural effusion 14. Most common benign ovarian tumour in women under the age of 25 years The correct answer is Dermoid cyst (teratoma) 15. The most common cause of ovarian enlargement in women of a reproductive age The correct answer is Follicular cyst Ovarian cysts: types